Physiology Block 2 practice questions and study guide
Ejecting blood at a reduced rate
A 14-year-old boy is in cardiac arrest after being hit with a baseball in the chest above the left ventricle. This devastating occurrence, called commotio cordis, is very rare, since the impact must occur during the ascending phase of the T wave. Which of the following events is also occurring at this time? Select one: a. Opening the mitral valve b. Rapidly filling the left ventricle c. Slowly filling the left ventricle d. Ejecting blood at a reduced rate e. Increasing left ventricular pressure
Factor VIII A young man with bleeding disorders and a history of bleeding disorders in the males of his family would lead one to suspect hemophilia A, a deficiency of factor VIII.
A 2-year-old boy bruises easily and has previously had bleeding gums. ! e maternal grandfather has a bleeding disorder. His physical examination shows several small bruises on the legs. Of which coagulation factor would you suspect this patient to be deficient? Select one: a. Factor VIII b. Factor X c. Prothrombin activator d. Factor II
Type B only Since the plasma contains antibodies the wrong plasma could contain antibodies against the B antigen. therefore, he could receive B plasma (containing anti-A antibodies) or AB (containing neither anti-A or anti-B antibodies.)
A 21-year-old female, blood type B, is undergoing surgery. Her platelet count is 75,000/!l. She will need platelet infusions before and during surgery. Which of the following blood types would be used to collect platelets that are compatible with the patient? Select one: a. Types B and O b. Type A only c. Type O only d. Type B only e. Types A, B, and AB only f. Types A and B g. Types A and B only
Low
(high/low) _____________ CARDIAC OUTPUT -> SNS activation + RAAS activation = ↑TPR + H2O and Na+ retention , increased ANP
vitamin K deficiency
decreased synthesis of factors II, VII, IX, X, protein C, protein S. Warfarin inhibits vitamin K epoxide reductase. Neonates lack enteric bacteria, which produce vitamin K. Early administration of vitamin K overcomes neonatal deficiency/coagulopathy.
First- degree heart block
defined as a higher-than-normal PR interval (greater than 0.2 seconds) generally asymptomatic and is characterized by a PR interval > 200 msec
Mobitz Type II heart block
diagnosed by unexpected, randomly dropped beats without lengthening or change in the PR interval. It is caused by fibrotic disease of the cardiac electrical conduction system, often as an acute or chronic complication of a myocardial infarction. It is symptomatic with occasional syncope and frequently progresses to third degree heart block. Therefore, pacemaker placement is indicated.
Phase 1
follows the closure of the fast Na+ channels, and the downward deflection is driven by a net outward current driven by potassium efflux from the cell.
hemophilias
hereditary bleeding disorders caused by lack of clotting factors Symptoms include prolonged bleeding and painful and disabled joints
Hemophilia C
mild type; factor XI deficiency
Hemophilia A
most common type of hemophilia (83% of all cases) due to a deficiency of factor VIII
Second-degree heart block
occurs when the action potential fails to reach the ventricles some, but not all, of the time
S4 heart sound
oscillations of the ventricles during atrial contraction •Recorded during atrial systole - ventricular diastole
phase 0
primarily driven by Na+ influx through the fast Na+ channels.
S3 heart sound
rapid ventricular filling phase •during transition between rapid filling and slow filling of ventricle - ventricular diastole
Sinus tachycardia
regular rhythm, above 100 beats/minute
Sinus bradycardia
regular rhythm, below 60 beats/minute
Afterload
the "load" that the heart must work against •aortic pressure - hypertension •TPR •aortic stenosis
extrinsic pathway the extrinsic pathway begins with the release of tissue thromboplastin in response to vascular injury or contact between traumatized extravascular tissue and blood. Tissue thromboplastin is composed of phos- pholipids from the membranes of tissue.
the coagulation pathway that begins with tissue thromboplastin is Select one: a. extrinsic pathway b. fibrin stabilization c. common pathway d. intrinsic pathway
preload
the load on the muscle in the relaxed state •load on ventricular muscle at the end of diastole •Left ventricular end-diastolic volume (LVEDV) •Left ventricular end-diastolic pressure (LVEDP) •↑ - Add volume , Slow heart rate, Constrict veins •↓ - Remove volume, Raise heart rate, Pool blood in veins
phase 3
the repolarization phase that is driven by potassium efflux, and corresponds to the T wave on a typical EKG tracing.
increased preload
venous return is increased end-diastolic volume is increased
increased contractility
ventricle can develop greater tension and pressure during systole
Third-degree (complete) heart block
when conduction of the action potential from the atria to the ventricles is completely interrupted. Under these conditions, pacemaker cells within the His-Purkinje system or the ventricular muscle cause the ventricles to beat at a low rate (idioventricular rhythm) independently of the atria there is no electrical communication between the atria and ventricles. The p-waves and QRS complexes on the electrocardiogram have no relationship to each other. Clinical symptoms are syncope, dizziness, and hypotension. Acute heart failure can result. A pacemaker must be urgently placed.
Torsades de pointes
• Polymorphic ventricular tachycardia •shifting sinusoidal waveforms
right sided heart failure
•"cor pulmonale" •Inability to pump blood to the pulmonary circulation -> ↑ RV pressure -> ↑ RA pressure -> ↑ systemic venous pressure (↑ jugular venous pressure) -> ↑ Pc -> lower extremity edema, liver congestion Hepatomegaly (nutmeg liver), Jugular venous distention, Peripheral edema•"cor pulmonale" •Inability to pump blood to the pulmonary circulation -> ↑ RV pressure -> ↑ RA pressure -> ↑ systemic venous pressure (↑ jugular venous pressure) -> ↑ Pc -> lower extremity edema, liver congestion Hepatomegaly (nutmeg liver), Jugular venous distention, Peripheral edema
Atrial flutter
•"sawtooth" appearance of the flutter waves
Wolff-Parkinson-White syndrome
•Abnormal fast accessory conduction pathway from atria to ventricle •shortened PR interval
Mitral valve prolapse
•Benign, asymptomatic •mid-systolic click and late systolic murmur
Hypokalaemia
•Flattened T wave •Large U wave
mitral regurgitation
•High left atrial pressure •Murmur: systolic, begins at S1 and continues to S2 pansystolic •↓ ESV, ↑ EDV , ↑ ventricular filling, ↑ SV, •No true isovolumetric phase •decompensated heart failure
aortic regurgitation
•Increased ventricular diastolic volume •Murmur: early diastolic; decrescendo, begins at S2. •chamber enlargement, hypertrophy, systolic dysfunction •Decreased aortic diastolic pressure •↑ SV, ↑ EDV, ↑ ESV
Mitral stenosis
•Left atrial pressure increases •associated with rheumatic fever •Increased pulmonary venous, capillary (edema, dyspnea), pulmonary arterial pressures •Murmur: diastolic; opening snap after S2, followed by a low-frequency decrescendo murmur (diastolic rumble) •↓ EDV, ↓ ESV , ↓ SV
Autoregulation
•The maintenance of a constant blood flow to an organ in the face of changing arterial pressure
Hyperkalaemia
•abnormally tall, tented T wave •potassium is responsible for repolarization of cardiac cells
Atrial fibrillation
•erratic baseline with no discrete P waves •Irregularly irregular heartbeat
aortic stenosis
•increases afterload •↓ SV, ↑ ESV , ↑ Left ventricular systolic pressure •Murmur: systolic; begins after S1 with a crescendo-decrescendo in intensity •Compensatory concentric hypertrophy develops •diastolic dysfunction •angina, exertional syncope, congestive heart failure
Sinus Arrhythmia
•rhythm is irregular and corresponds to the respiratory cycle
Polycythemia
↑ RBC count, Htc and Hb concentration (Hypoxic erythropoietic drive - Secondary Polycythemia, Polycythemia vera)
2 The intersection of cardiac output and venous return curves represents the equilibrium function of the heart. In a normal person, exercise will cause an increase in both cardiac output and venous return leading to a new equilibrium at point 2. This patient is a healthy athlete with a provided equilibrium at point 1. During exercise, increased contractility (+ inotropy) leads to increased stroke volume and heart rate resulting in increased cardiac output. This is represented by a shift of the cardiac output curve up and to the left. Illustration A shows the annotated completion of Figure A. Notice that the positive inotropy curve in green and the positive venous return curve in red represent the physiological increase in blood flow during exercise. Incorrect Answers: Answer 1: This is the resting state for a healthy individual. The cardiovascular system undergoes changes during exercise in healthy individuals as explained in the discussion. Answer 3: This state represents an increase in TPR and a decrease in overall cardiac output. This can be seen in the acute stages of massive hemorrhage with decompensated shock physiology. Answer 4: This state represents negative inotropy. This can happen with narcotics that increase venous capacitance, decreasing mean circulatory filling pressure, and decreasing venous return. Answer 5: This is the x-intercept of the graph and represents mean circulatory filling pressure
A 21-year-old male comes into the physician for a yearly physical. He is a swimmer with no significant past medical history. His baseline cardiac output and venous return curves are shown in Figure A. Assuming that he starts at position 1, which of the following points would most likely represent his cardiac output and venous return curves during one of his swim practices? Select one: a. 2 b. 1 c. 4 d. 5 e. 3
A The arterial baroreceptors are activated in response to a fall in arterial pressure. During hemorrhage, the fall in arterial pressure at the level of the baroreceptors results in enhanced sympathetic outflow from the va- somotor center and a decrease in parasympathetic nerve activity. e increase in sympathetic nerve activ- ity leads to constriction of peripheral blood vessels, increased total peripheral resistance, and a return of blood pressure toward normal. e decrease in para- sympathetic nerve activity and sympathetic outflow would result in an increase in heart rate.
A 22-year-old man enters the hospital emergency room after severing a major artery in a motorcycle accident. It is estimated that he has lost approximately 700 ml of blood. His blood pressure is 90/55 mm Hg. Which of the following sets of changes would be expected in response to hemorrhage in this man? Select one: a. B b. D c. A d. H e. C f. G g. F h. E
0.50 mm Hg/ml/min Vascular resistance = arterial pressure - venous pressure / blood flow. In this example, arterial pressure is 130 mm Hg, venous pressure is 5 mm Hg, and blood flow is 250 ml/min. THus, vascular resistance 125 / 250, or 0.50 mm Hg/ml/min.
A 22-year-old man has a muscle blood flow of 250 ml/ min and a hematocrit of 50. He has a mean arterial pressure of 130 mm Hg, a muscle venous pressure of 5 mm Hg, and a heart rate of 80 beats/min. Which of the following is the approximate vascular resistance in the muscle of this man? Select one: a. 1.00 mm Hg/ml/min b. 0.20 mm Hg/ml/min c. 0.50 mm Hg/ml/min d. 2.50 mm Hg/ml/min e. 0.10 mm Hg/ml/min
Increased central venous pressure Net filtration from systemic capillaries is dependent on the Starling forces and capillary permeability. The equation is Net filtration = Kf [(Pcapillary - Ptissue) - (πcapillary - πtissue)], where Kf is the filtration coefficient of the membrane, and is directly proportional to capillary permeability, Pcapillary and Ptissue are the hydrostatic pressures in the capillary and tissue (interstitial space), respectively, and πcapillary and πtissue are the osmotic (colloid oncotic) pressures in the capillary and interstitial space, respectively. Increasing central venous pressure increases the capillary hydrostatic pressure (Pcapillary), which increases the filtration of fluid from the systemic capillaries, leading to edema. All of the other choices will cause a decrease in filtration.
A 22-year-old woman is hospitalized with a history of respiratory distress, fever, and fatigue. ST segment and T wave abnormalities suggest myocarditis, which is attributed to an acute viral origin. Over the next several days, significant peripheral edema develops. The edema is most likely caused by which of the following? Select one: a. Increased plasma protein concentration b. Decreased capillary permeability c. Decreased arterial pressure d. Increased lymphatic flow e. Increased central venous pressure
Valsalva maneuver This patient has a tall, slender physique and a family history of a father who died at age 53 from an aortic dissection. Her appearance and family history suggest Marfan syndrome. [Other findings that would be suggestive of Marfan syndrome include arachnodactyly (long and slender fingers, shown in the image) and ocular lens dislocation. Marfan syndrome is associated with a high prevalence of mitral valve prolapse (MVP) is classically described as a midsystolic click followed by a crescendo-decrescendo murmur heard best at the apex. Maneuvers that decrease venous return (eg, Valsalva maneuver, standing) and volume of the left ventricle worsen the prolapse. When these maneuvers are performed, the crescendo-decrescendo murmur heard in MVP increases in intensity and duration, and the midsystolic click occurs earlier. Aortic insufficiency (AI) is another valvular lesion that may occur in patients with Marfan syndrome, secondary to aortic root dilatation. AI would manifest with a diastolic decrescendo murmur that is louder on expiration and accentuated by hand grip.] Inspiration increases venous return to the right atrium and would potentially decrease murmur intensity. Lying down increases venous return to the right atrium and could decrease the murmur. The Müller maneuver involves asking the patient to inhale against a closed mouth and nose, and it can be used to detect collapsed upper airways. Rapid squatting heightens preload by increasing the return of blood to the heart and boosts afterload via constriction of peripheral vessels. This decreases the MVP murmur.
A 22-year-old woman who is a medical student presents to a physician as a new patient. During the initial visit, the student notes that she has no health complaints. The patient's family history includes a healthy mother and brother, but her father died at age 53 from an aortic dissection. Physical examination indicates that the patient is 187.96 cm (6'2") tall, and weighs 61 kg (135 lb). Physical examination reveals chest asymmetry. Pulse is 64/min and a blood pressure is 115/70 mm Hg. An auscultation presents a midsystolic click followed by a crescendo-decrescendo murmur heard best at the apex Which of the following physical maneuvers can the patient perform to accentuate the cardiac auscultation findings? Select one: a. Inspiration b. Rapid squatting c. Valsalva maneuver d. Lying down e. Müller maneuver
Anemia The magnitude of the cardiac output is regulated to maintain an adequate blood pressure and to deliver an adequate supply of oxygen to the tissues. In anemia, a greater cardiac output is required to supply oxygen to the tissues because the oxygen-carrying capacity of the blood is reduced. The reduced blood viscos- ity increases the velocity and thus the turbulence of the blood flow, which makes systolic murmurs common in anemic patients. In aortic regurgitation, the stroke volume will be increased, but a portion of the blood ejected by the heart will return to the heart during diastole. Thus, the output delivered to the tissues does not increase despite the fact that the blood ejected by the heart has increased. In hypertension, third-degree heart block, and cardiac tamponade (decreased filling of the heart due to accumulation of fluid within the pericardium), cardiac output will be normal or reduced.
A 23-year-old female presents with fatigue and is found to have a systolic murmur and higher than normal cardiac output. The differential diagnosis based on these findings includes which of the following? Select one: a. Anemia b. Aortic regurgitation c. Hypertension d. Mitral valve prolapse e. Mitral stenosis
A In patent ductus arteriosus, a large quantity of the blood pumped into the aorta by the left ventricle immediately flows backward into the pulmonary artery and then into the lung and left atrium. e shunting of blood from the aorta results in a low diastolic pressure, while the increased inflow of blood into the left atrium and ventricle increases stroke volume and systolic pressure. e combined increase in systolic pressure and decrease in diastolic pressure results in an increase in pulse pressure.
A 24-year-old woman delivers a 6-lb, 8-oz female baby. The newborn is diagnosed as having patent ductus arteriosus. Which of the following sets of changes would be expected in this baby? Select one: a. A b. G c. E d. H e. F f. D g. B h. C
Delayed closing of the pulmonic valve The second heart sound (S2) is associated with the closing of the aortic and pulmonic valves. The aortic valve normally closes slightly before the pulmonic valve, resulting in a splitting of the second heart sound into two components. During inspiration, the closing of the pulmonic valve is delayed, resulting in a prolongation of the interval between the two components of the second heart sound. During inspiration, the preload on the right heart is increased, resulting in a larger stroke volume, which causes the delay in the closing of the pulmonic valve. Closure of the mitral valve and tricuspid valves are associated with the first heart sound (S1). Heart rate is higher during inspiration than during expiration.
A 24-year-old woman undergoes an annual physical examination for participation on the varsity track team at her college. While auscultating her heart sounds, the sports medicine physician instructs the woman to take in a deep inspiration. During this maneuver, he detects splitting of the second heart sound. Which of the following is the mechanism underlying this finding? Select one: a. A decrease in heart rate b. An increased left ventricular stroke volume c. Delayed opening of the mitral valve d. Delayed closing of the aortic valve e. Delayed closing of the pulmonic valve
Polymorphonuclear cells
A 29-year-old man presents to the doctor's office with cough, fever, malaise and chills. A bacterial pneumonia is diagnosed and the patients is prescribed antibiotics. Which of the following cells will most likely be responsible for phagocytosis of the bacterial pathogens causing this patient's pneumonia? Select one: a. Lymphocytes b. Mast cells c. Polymorphonuclear cells d. Eosinophils e. Platelets
Megaloblastic Anemia Anemia - Deficiency of blood Hb due to ↓ RBC count (too rapid loss or/and too slow production) or ↓ Hb quantity in the RBC Results in ↓ oxygen-carrying capacity of the blood → hypoxia → vasodilation ↑ in pulse and respiratory rates (effort to supply sufficient oxygen to tissues) ↓ exercise & cold tolerancePale skin (↓ red colored HbO) ↑ fatigue and lassitude (lack of energy) ↓ blood viscosity → ↓ peripheral resistance → ↑ blood flow, venous return and CO and work load on the heart •Deficiency of vitamin B12, folic acid or IF (i.e., atrophic gastric mucosa, pernicious anemia) - megaloblastic anemia •Maturation failure •→ ↓ rate of erythroblasts proliferation •→ production of large precursor cells - megaloblasts and larger irregular oval erythrocytes - macrocytes fully saturated with Hb - macrocytic (megaloblastic) anemia •→ ↑ fragility of the plasma membrane → ↓ life span → anemia.
A 30 years old male is brought to the hospital with history of gastrectomy. His skin appears lemon-yellow.Investigations reveal hemoglobin 10 g/dl, odd shaped RBCs and Serum Vitamin B12 is low. He is likely to be suffering from: Select one: a. Microcitic anemia b. Aplastic Anemia c. Blood Loss Anemia d. Megaloblastic Anemia e. Hemolytic Anemia
Atrial septal defect This patient presents with a stroke in the left parietal lobe and widely patent carotid arteries. Auscultation reveals a midsystolic murmur with a widely-split and fixed S2.. He most likely has an atrial septal defect (ASD). This pathology allowed a paradoxical embolus from the right side of the heart to pass into the left side, and then embolize in the left middle cerebral artery. The embolus is only able to pass from right to left atria when there is a transient reversal of the left to right flow that normally occurs with an ASD. This transient reversal occurs during coughs, bearing down for bowel movements, or anything that replicates the Valsalva maneuver. In normal physiology, inspiration leads to increased blood in the right ventricle, causing delay in the closure of the pulmonic valve. Expiration decreases blood in the right ventricle, leading to earlier closure of the pulmonic valve. However, with an ASD, a certain amount of blood passes from the left atrium to the right atrium on every beat. This decreases the variation in pulmonary vasculature on respiration, causing a wide and "fixed" S2 heard on auscultation.Aortic valve vegetation occurs in the setting of infection of the aortic valve. Atrial fibrillation could cause a left atrial thrombus. Patent ductus arteriosus is characterized by a machine-like continuous murmur. Pulmonary stenosis produces a systolic ejection murmur at the left second intercostal space, but does not present with fixed-splitting. A ventricular septal defect is often symptomatic from a young age; this patient has no history of heart malformations of any kind.
A 35-year-old man comes the emergency department with a 1-hour history of difficulty speaking and weakness in his right arm. MRI of the head and neck confirms a stroke in the left parietal lobe and widely patent carotid arteries. There is no history or ECG changes suggestive of atrial fibrillation. His wife, who brought him to the hospital, reports that he returned this morning from a business trip in Europe. When she picked him up from the airport, he was completely fine. Auscultation of the chest reveals the a midsystolic murmur with a widely-split and fixed S2Which of the following best accounts for this patient's stroke? Select one: a. Pulmonary stenosis b. Patent ductus arteriosus c. Ventricular septal defect d. Atrial septal defect e. Aortic valve vegetation f. Left atrial thrombus
E Pulse pressure is the difference between systolic pressure and diastolic pressure. e two major factors that affect pulse pressure are the stroke volume output of the heart and the compliance of the arterial tree. An increase in stroke volume increases systolic and pulse pressure, while an increase in compliance of the arterial tree decreases pulse pressure. Moderate aortic valve stenosis results in a decrease in stroke volume, which leads to a decrease in systolic pressure and pulse pressure.
A 35-year-old woman visits her family practice physician for an examination. She has a mean arterial blood pressure of 105 mm Hg and a heart rate of 74 beats/ min. Further tests by a cardiologist reveal that the patient has moderate aortic valve stenosis. Which of the following sets of changes would be expected in this patient? Select one: a. D b. A c. E d. C e. F f. G g. B h. H
A The difference between systolic pressure and diastolic pressure is the pulse pressure. e two major factors that affect pulse pressure are the stroke volume output of the heart and the compliance of the arterial tree. In patients with moderate aortic regurgitation (due to incomplete closure of aortic valve), the blood that is pumped into the aorta immediately flows back into the left ventricle. e backflow of blood into the left ventricle increases stroke volume and systolic pressure. e rapid backflow of blood also results in a decrease in diastolic pressure. us, patients with moderate aortic regurgitation have high systolic pressure, low diastolic pressure, and high pulse pressure.
A 35-year-old woman visits her family practitioner for an examination. She has a blood pressure of 160/75 mm Hg and a heart rate of 74 beats/min. Further tests by a cardiologist reveal that the patient has moderate aortic regurgitation. Which of the following sets of changes would be expected in this patient? Select one: a. D b. A c. B d. G e. E f. F g. C h. H
Cardiac output is higher than normal Distributive shock is char- acterized by a decreased peripheral resistance and a high cardiac output. The decrease in peripheral resistance is so great that mean blood pressure falls below normal. Hypovolemic shock is caused by a decrease in circulat- ing blood volume and pressure. In both types of shock, the baroreceptor reflex increases ventricular contractility and heart rate. Also, in both types of shock, blood is shunted away from the kidney, decreasing renal blood flow and glomerular filtration. As a result, plasma creatinine levels rise.
A 37-year-old patient is brought to the Emergency Department in shock. Which of the following is a reason to direct treatment toward destributive shock rather than hypovolumic shock? Select one: a. Ventricular contractility is greater than normal b. Heart rate is greater than normal c. Serum creatinine is elevated d. Total peripheral resistance is greater than normal e. Cardiac output is higher than normal
Aortic valve regurgitation the blowing murmurs of relatively high pitch are usually the murmurs associated with valvular insufficiency.the key pieces of data to identify this murmur are the systolic and diastolic pressures. Aortic valve regurgitation typically has a high pulse pressure, which is the systolic - the diastolic pressure, and in this case is 100 mm Hg. Also note that the diastolic pressure decreases to very low values of 40 mm Hg as the blood leaks back into the left ventricle.
A 40-year-old woman has been diagnosed with a heart murmur. A "blowing" murmur of relatively high pitch is heard maximally over the left ventricle. " e chest x-ray shows an enlarged heart. Arterial pressure in the aorta is 140/40 mm Hg. What is the diagnosis? Select one: a. Aortic valve stenosis b. Tricuspid valve regurgitation c. Aortic valve regurgitation d. Mitral valve stenosis e. Pulmonary valve stenosis
Hemolytic anemia this patient has decreased production of red blood cells as confirmed by the anemia (low number, Hb, and Hct), yet the red blood cells being produced have a normal size, MCV 90. erefore, the patient does not have spherocytosis (small red cells) or vitamin B12 deficiency (large red cells). the normal WBC count and the increased reticulocyte count suggest that the bone marrow is functioning. the increased reticu- locyte count means that a large number of red cells are being produced. ese laboratory values support an anemia due to some type of blood loss; in this case an anemia due to hemolysis.
A 40-year-old woman visits the clinic complaining of fatigue. She had recently been treated for an infection. Her laboratory values are RBC 1.8 x 106 /ml, Hb 5.2 g/dL, hematocrit (Hct) 15, WBC 7.6 x 103 /l, platelet count 320,000/ml, mean corpuscular volume (MCV) 92 fL, and reticulocyte count 24%. What is the most likely explanationfor this presentation? Select one: a. B12 deficiency b. Aplastic anemia c. Hereditary spherocytosis d. Hemolytic anemia
IL-3
A 44-year-old woman is being treated for cancer and develops neutropenia. The physician is considering administration of a growth factor to stimulate leucopoiesis. First, he decides to test the level of a naturally occurring cytokine necessary to stimulate all bone marrow cell lines. Which of the following choices should he test for? Select one: a. IL-4 b. IL-5 c. IL-7 d. IL-1 e. erythropietin f. IL-6 g. IL-3 h. IL-2
End-stage renal disease this patient is anemic, but the RBCs being pro- duced are normal (note normal MCV). e overall production of the RBCs is decreased (reticulocyte count is low). WBCs and platelets are normal, suggest- ing a normal bone marrow. Folic acid and iron deficiency anemia would result in a lower RBC MCV. Hemolytic anemia would result in an increase reticulocyte count. e elevated blood pressure provides evidence of renal disease. is patient has end-stage renal disease and decreased erythropoietin production.
A 45-year-old woman developed fatigue in July and had blood counts that were reported to be normal. She washospitalized because of a very severe headache in December, and was found to have a blood pressure of 175/90. Her laboratory values were as follows: hemoglobin (8.3 g/dL), RBC count (2.2 x 106 /ml), Hct (23%), MCV (89 fL), WBCs (5100/! l), platelets (262 x 103 /ml), and reticulocyte count 0.8%. What is the diagnosis for this patient? Select one: a. Iron deficiency b. End-stage renal disease c. Folic acid deficiency d. Hemolytic anemia
A decreased pulse pressure In aortic stenosis, the resistance of the aortic valve increases, making it more difficult for blood to be ejected from the heart. Because a pressure drop occurs across the stenotic aortic valve, the ventricular pressure is much larger than the aortic pressure. Although stroke volume typically decreases leading to a decrease in pulse pressure, a normal cardiac output and arter- ial pressure can still be maintained by increasing heart rate. However, the increased afterload will lead to a decreased ejection fraction and increased cardiac oxygen consumption.
A 47-year-old female is brought to the Emergency Department because she fainted at the gym during her daily aerobic workout. A prominent systolic murmur is heard and a presumptive diagnosis of aortic stenosis is made. Which of the following is consistent with that diagnosis? Select one: a. A decreased left ventricular diastolic pressure b. An increased ejection fraction c. A decreased cardiac oxygen consumption d. An increased arterial pressure e. A decreased pulse pressure
Phase 0, which is primarily characterized by sodium influx
A 48-year-old female comes into the ER with chest pain. An electrocardiogram (EKG) shows a heart beat of this individual in Image A. The QR segment best correlates with what part of the action potential of the ventricular myocyte shown in Image B? Select one: a. Phase 0, which is primarily characterized by potassium efflux b. Phase 1, which is primarily characterized by potassium and chloride efflux c. Phase 1, which is primarily characterized by calcium efflux d. Phase 3, which is primarily characterized by potassium efflux e. Phase 0, which is primarily characterized by sodium influx
1/16
A 48-year-old man presents to the emergency department with chest pain that radiates to his jaw and left shoulder. Angina pectoris is suspected, and he is sent for an angiogram. The test reveals an atherosclerotic coronary artery that is 50% occluded. The maximal blood flow through this artery is reduced by Select one: a. 1/16 b. 1/2 c. 1/8 d. 1/32 e. 1/4
Aortic valve stenosis this patient has a QRS axis of -45°, indicating a leftward axis shift. In other words, the left side of the heart is enlarged. In aortic valve stenosis the left side of the heart is enlarged because of the extra tension the left ventricular walls must exert to expel blood out the aorta. "erefore, these symptoms fit with a patient with aortic stenosis. In pulmonary valve stenosis, the right side of the heart hypertrophies, and in mitral valve stenosis there is no left ventricular hypertrophy. In tricuspid valve regurgitation the right side of the heart enlarges, and in tricuspid valve stenosis no ventricular hypertrophy occurs.
A 50-year-old female patient at UMC has been diagnosed with a heart murmur. A murmur of relatively low pitch is heard maximally over the second intercostal space to the right of the sternum. The chest x-ray shows an enlarged heart. " e mean QRS axis of the EKG is -45°. The diagnosis is Select one: a. Mitral valve stenosis b. Tricuspid valve regurgitation c. Tricuspid valve stenosis d. Aortic valve stenosis e. Pulmonary valve stenosis
E
A 50-year-old male patient with liver cancer is in advanced-stage hepatic failure. What effect will this have on his capillary colloid osmotic pressure, his interstitial hydrostatic pressure, and the net filtration in his systemic capillaries? Select one: a. A b. B c. C d. D e. E f. F
point U
A 50-year-old man visits his physician for an annual exam. He complains of mild breathlessness on exertion. Upon cardiac auscultation the physician detects an S3 heart sound. At which point on the cardiac cyclediagram does the S3 heart sound occur? Select one: a. R b. U c. P d. T e. Q f. S
Anemia The third heart sound (S3) is associated with rapid ventricular filling of the left ventricle. S3 is commonly heard in children and young adults and is also heard in conditions when rapid filling of the ventricle is abnormally high, for example, in anemia or mitral regurgitation. It can also occur in dilated car- diomyopathy, restrictive cardiomyopathy, or cardiac tamponade when the compliance of the left ventricle is reduced. An increased heart rate reduces the duration of diastole, and thus tends to decrease ventricular filling.
A 55-year-old female presents for her annual physical examination. Upon auscultation, a third heart sound is heard. The differential diagnosis of this finding includes which of the following? Select one: a. Aortic regurgitation b. Anemia c. Mitral stenosis d. Tachycardia e. Right bundle branch block
Third-degree heart block
A 55-year-old male reports several recent episodes of syncope. An electrocardiogram is performed. Which of the following arrhythmias is most commonly associated with syncope? Select one: a. First-degree heart block b. Tachycardia c. Sinus arrhythmia d. Third-degree heart block e. Second-degree heart block
Increased left atrial pressure Increased rather than decreased left atrial pressure would be seen in a patient with mitral stenosis as a result of a pressure gradient it creates between the left ventricle and the left atrium. An overall decrease in left ventricular pressure would not be seen in a patient with mitral valve stenosis. The decrease only occurs relative to the left atrium. Increased left ventricular pressure would be seen in patients with aortic stenosis. Increased right atrial pressure occurs in response to pulmonic valve stenosis or tricuspid valve stenosis.
A 56-year-old man comes to his primary care physician for a check-up for the first time in 3 years. His medical history is significant for hypercholesterolemia and type 2 diabetes mellitus, as well as for an acute illness at the age of 9 years. The childhood illness involved a high fever, pleuritic chest pain, migrating joint pain, and a pink, nonpruritic rash on his torso. His vital signs include a blood pressure of 135/85 mm Hg and heart rate of 70. Auscultation of his heart reveals a low-pitched diastolic rumble heard best at the apex. No abnormal findings are detected by echocardiography. Of the following, which is the most likely change in pressure that would be seen in this patient? Select one: a. Decreased left atrial pressure b. Decreased left ventricular pressure c. Increased right atrial pressure d. Increased left atrial pressure e. Increased left ventricular pressure
Hypersegmented Neutrophils
A 56-year-old man with a history of Crohn's Disease presents to the doctor's office for follow up after a recent surgery removing a portion of his terminal ileum. The patient complains of mild symptoms of weakness and parasthesias. The physician decides to perform a blood smear. Which of the following changes is expected to be seen on the smear that would be consistent with the patient's symptoms? Select one: a. Normocytic anemia b. Microcytic anemia c. Hypersegmented Neutrophils d. Thrombocytopenia e. Hypochromasia
An increased left atrial pressure Atrial fibrillation is an arrhythmia in which the electrical activity of the atrium becomes disorga- nized and therefore unable to produce a coordinated atrial contraction. The absence of an atrial pulse reduces the emptying of the atria during diastole and results in an enlarged left atrium and increased left atrial pressure. The venous A wave represents atrial contraction and disappears due to the absence of an atrial beat. Decreased filling of the heart results in a decrease in stroke volume. Heart rate increases because the continuous electrical activity of the atria ini- tiates a high rate of ventricular activity. Systemic blood pressure typically falls because of inadequate filling of the ventricles and the resulting decrease in stroke volume.
A 57-year-old male complains of an irregular heart beat that he notices is relieved by pressing on his eyeball. An electrocardiogram reveals atrial fibrillation. Which of the following is most likely to accompany this condition? Select one: a. An increased mean arterial blood pressure b. An increased venous A wave c. An increased left atrial pressure d. An increased stroke volume e. A decreased heart rate
B Point A corresponds to the opening of the mitral valve at the beginning of diastole the line from A to C shows the increase in ventricular volume during diastole Point C marks the beginning of systole as left ventricular pressure becomes greater than left atrial pressure, causing the mitral valve to close. This closure (in conjunction with the closure of the tricuspid valve) represents S1. The line from point C to D corresponds to isovolumetric contraction, during which both the mitral and aortic valves remain closed as the left ventricular pressure increases. At point D the left ventricular pressure becomes greater than the aortic pressure and the aortic valve opens. Between points D and F, the left ventricular pressure continues to increase as the ventricle continues to contract and blood is ejected from the left ventricle into the aorta. At point F the aortic valve closes when the left ventricle begins to relax and the left ventricular pressure becomes less than aortic pressure. This closure (in conjunction with the closure of the pulmonic valve) represents S2. The line from point F to point A represents the isovolumetric relaxation that occurs after the end of ventricular systole. When the left ventricular pressure becomes less than the pressure in the left atrium, the mitral valve opens, thus beginning a new loop of the cardiac cycle (diastole plus systole). Point B corresponds to the point near the end of diastole when S4 may be heard. An S4 (which occurs when a stiff ventricle causes turbulent flow in the atria), commonly follows the "atrial kick" (forceful atrial systole, a cause of the S4). It is not normally present in adults. Its presence suggests a decrease in ventricular compliance, such as occurs in ventricular hypertrophy resulting from chronic hypertension. S4 is thought to result from vibration of a stiff, noncompliant ventricular wall as blood is rapidly ejected into the ventricle.
A 58-year-old man with a past medical history of hypertension goes to his physician for a routine visit. On physical examination the physician is able to detect an S4 heart sound, and refers the patient to a cardiologist. After a thorough work-up, he is found to have left ventricular hypertrophy. The image below plots left ventricular pressure versus left ventricular volume for a single cardiac cycle.This patient's S4 heart sound heard on auscultation would best correspond to which of the following points? Select one: a. A b. F c. D d. C e. B f. E
Mobitz type II heart block
A 59-year-old man presents to emergency department after two episodes of loss of consciousness without an apparent cause. His wife, who witnessed the episodes, states that before each episode, he became slightly pale, suddenly fell to the ground, and remained unresponsive for about one minute. He did not move or shake, and his breathing continued normally. The patient has a history of hypertension, coronary artery disease, and an anterior myocardial infarction six months ago that required a stent placement. He takes hydrochlorothiazide, lisinopril, metoprolol, and aspirin. He has no family history of neurologic or cardiovascular diseases. The medical workup includes basic laboratory testing and an electrocardiogram, which is shown Which of the following conditions is represented by the electrocardiogram? Select one: a. Sinus bradycardia b. Mobitz type I heart block c. First degree heart block d. Mobitz type II heart block e. Third degree heart block
Factor VIII Hemophilias are hereditary bleeding disorders caused by lack of clotting factorsHemophilia A - most common type (83% of all cases) due to a deficiency of factor VIIIHemophilia B - results from a deficiency of factor IXHemophilia C - mild type, caused by a deficiency of factor XISymptoms include prolonged bleeding and painful and disabled jointsHemophilia A and B are sex linked (X chromosome)Usually occurs in males Females are hemophilia carriers. Only under rare circumstances females actually suffer from hemophilia
A 6 year old boy bruises easily and has previously bleeding gums. The maternal grandfather also has a bleeding disorder. You suspect the deficiency of Select one: a. Factor X b. Factor II c. Factor VIII d. Prothrombin activator e. Factor XIII
B A person with atherosclerosis would be expected to have decreased arterial compliance. e decrease in arterial compliance would lead to an increase in sys- tolic pressure and pulse pressure.
A 60-year-old man visits his family practitioner for an annual examination. He has a mean blood pressure of 130 mm Hg and a heart rate of 78 beats/min. His plasma cholesterol level is in the upper 25th percentile, and he is diagnosed as having atherosclerosis. Which of the following sets of changes would be expected in this patient? Select one: a. H b. A c. B d. E e. F f. C g. D h. G
Third-degree A-V heart block By definition, first-degree A-V heart block occurs when the P-R interval exceeds a value of 0.20 sec, but without any dropped QRS waves. In the following figure, the P-R interval is about 0.30 sec, which is con- siderably prolonged. However, there are no dropped QRS waves. During second-degree A-V block or third- degree A-V block, QRS waves are dropped.
A 60-year-old man weighing 220 lb had the following EKG, which shows the standard lead II. What is his diagnosis? Select one: a. First-degree A-V heart block b. Atrial flutter c. Second-degree A-V heart block d. A-V nodal rhythm e. Tird-degree A-V heart block
-50° The mean electrical axis can be determined plot- ting the resultant voltage of the QRS for leads I, II, and III. e result is as is shown above and has a value of 50°.
A 60-year-old woman had the following EKG recorded at a local emergency room following an automobile accident. Her weight was 70 kg and her aortic blood pressure was 140/80 mm Hg. What is the mean electrical axis calculated from standard leads I, II, and III shown in her EKG? Select one: a. +100 b. -50° c. -12° d. +170 e. -90°
Ventricular rate of contraction is 140 beats/min Atrial fibrillation has a rapid irregular heart rate. e P waves are missing or are very weak. e atria exhibit circus movements, and often are very enlarged, causing the atrial fibrillation.
A 60-year-old woman has been diagnosed with atrial fibrillation. Which of the following statements best describe this condition? Select one: a. P waves of the EKG are pronounced b. Atria are smaller than normal c. Ventricular contractions occur at regular intervals d. Ventricular rate of contraction is 140 beats/min e. QRS waves are more pronounced than norma
Left bundle branch block The patient has a left axis deviation because of the large negative deflection of the R wave in lead III. Also, her T wave was inverted in lead I, which means that it is in the opposite direction of the QRS complex. is is characteristic of bundle branch block. Also, the QRS complex had a width of 0.20 sec, a very prolonged QRS complex. A QRS complex that has a width greater than 0.12 sec is normally caused by a conduction block. All these factors indicate that this patient has a left bundle branch block.
A 60-year-old woman has lost some ability to perform normal household tasks and is not feeling well. An EKG shows a QRS complex with a width of 0.20 sec, the T wave is inverted in lead I, and the R wave has a large negative deflection in lead III. Which of the following is the likely diagnosis? Select one: a. Right bundle branch block b. Left ventricular hypertrophy c. Pulmonary valve stenosis d. Left bundle branch block e. Right ventricular hypertrophy
Atrial fibrillation First-, second-, and third-degree heart blocks as well as atrial paroxysmal tachycardia all have P waves in the EKG. However, there are usually no evident P waves during atrial fibrillation, and the heart rate is irregular. erefore, this EKG is characteristic of atrial fibrillation.
A 60-year-old woman sees her physician for her annual physical examination. !e physician orders an EKG, which is shown below. Which of the following is the likely diagnosis? Select one: a. Second-degree A-V block b. Atrial paroxysmal tachycardia c. third-degree A-V block d. First-degree A-V block e. Atrial fibrillation
Pulmonary hypertension The EKG from this patient has a positive deflection in aVF and a negative deflection in standard limb lead I. erefore, the mean electrical axis is between 90° and 180°, which is a rightward shift in the EKG mean electrical axis. Systemic hypertension, aortic valve stenosis, and aortic valve regurgitation cause hypertrophy of the left ventricle and thus a leftward shift in the mean electrical axis. Pulmonary hyperten- sion causes a rightward shift in the axis, and is there- fore characterized by this EKG.
A 60-year-old woman tires easily. Her EKG shows a QRS complex that is positive in the aVF lead and negative in standard limb lead I. A likely cause of this condition is which of the following? Select one: a. Chronic systemic hypertension b. Aortic valve regurgitation c. Aortic valve stenosis d. Pulmonary hypertension
polycythemia vera the increase in RBC, WBC, and platelets suggests that the patient is suffering from polycythemia vera. Renal disease would result in a low EPO level, but the RBC count would be low. Myeloid leukemia would result in an increase in WBCs, with no increase in RBCs. Secondary polycythemia would have an elevated EPO level. Relative is due to dehydration.
A 62-year-old female arrives for her annual physical. She complains of itching in her hands along with headaches and vertigo. A routine complete blood count (CBC) shows red blood cells (RBCs) of 8.2 million/ml, white blood cells (WBCs) 37,000/ml, and platelets 640,000/ml. Her erythropoietin levels are lower than normal. the primary diagnosis would be Select one: a. secondary polycythemia b. polycythemia vera c. myeloid leukemia d. relative polycythemia e. thrombocytopenia
A high velocity of blood within the carotid artery
A 63-year-old woman presented with acute onset of right eye pain. Ophthalmic and neurologic examinations were normal except for a loud right carotid bruit. The eye pain ceased following carotid endarterectomy (a surgical procedure to remove the atheromatous plaque material, or blockage, in the lining of an artery). The bruit was most likely caused by which of the following? Select one: a. Widening of the carotid artery b. An increase in blood viscosity c. A high velocity of blood within the carotid artery d. An increase in hematocrit e. Lengthening of the carotid artery
Warfarin this clot is due to stasis of blood flow in her venous circulation. Heparin is used for the prevention of a clot, but has to be infused. is anticoagulation occurs by heparin binding to antithrombin III and the subsequent inactivation of thrombin. A continuous heparin drip is impractical. Warfarin is used to inhibit the formation of vitamin K clotting factors and would prevent the formation of any clot. Aspirin is used to prevent activation of platelets. e current clot is not due to activation of platelets. Vitamin K will be used to restore clotting factors that may be decreased after warfarin treatment. is patient has sufficient clotting factors as evidenced by her venous clot.
A 63-year-old woman returned to work following a vacation in New Zealand. Several days after returning home, she awoke with swelling and pain in her right leg and her leg was blue. She went immediately to the emergency room where an examination showed an extensive deep vein thrombosis involving the femoral and iliac veins on the right side. Following resolution of the clot, this patient will require which treatment in the future? Select one: a. Vitamin K b. Warfarin c. Continual heparin infusion d. Aspirin Feedback
Warfarin this clot is due to stasis of blood flow in her venous circulation. Heparin is used for the prevention of a clot, but has to be infused. is anticoagulation occurs by heparin binding to antithrombin III and the subsequent inactivation of thrombin. A continuous heparin drip is impractical. Warfarin is used to inhibit the formation of vitamin K clotting factors and would prevent the formation of any clot. Aspirin is used to prevent activation of platelets. e current clot is not due to activation of platelets. Vitamin K will be used to restore clotting factors that may be decreased after warfarin treatment. is patient has sufficient clotting factors as evidenced by her venous clot.
A 63-year-old woman returned to work following a vacation in New Zealand. Several days after returning home, she awoke with swelling and pain in her right leg and her leg was blue. She went immediately to the emergency room where an examination showed an extensive deep vein thrombosis involving the femoral and iliac veins on the right side. Following resolution of the clot, this patient will require which treatment in the future? Select one: a. Vitamin K b. Aspirin c. Warfarin d. Continual heparin infusion
Decreasing preload Ischemic cardiac pain is produced when oxygen demand is greater than the oxygen that can be delivered by the coronary arteries. Under normal circumstances an increase in contractility will increase oxygen demand, but in a dilated heart, an increase in contractility actually decreases oxygen demand. In heart failure, positive inotropic agents reduce oxygen demand by reducing end- diastolic volume (preload) and thus the wall stress that must be developed by the heart with each beat. Reducing volume decreases wall stress because, according to the law of Laplace, the wall stress is proportional to the product of force and radius (which is proportional to ventricular volume).
A 63-year-old woman with congestive heart failure is given digitalis. Positive inotropic drugs can reduce ischemic cardiac pain (angina) in a dilated failing heart by doing which of the following? Select one: a. Increasing diastolic filling time b. •Increasing coronary blood flow c. Decreasing preload d. Increasing heart rate e. Decreasing total peripheral resistance
Prolonged systolic ejection The patient presents with a systolic murmur with ejection click is most indicative of aortic stenosis. The ejection click occurs due to a stiff, calcified valve opening with very high pressures. With a calcified vessel there is delayed closure of the aortic valve, with very high pressure and turbulent flow through the stenotic valve. The location of the murmur and radiation to the neck are classic for aortic stenosis. This outflow obstruction leads to an increase in left ventricular (LV) systolic pressure, as demonstrated in the diagram. As a compensatory mechanism to normalize LV wall stress, LV wall thickness increases by parallel replication of sarcomeres, producing concentric hypertrophy. Increased LV mass and increased systolic pressure lead to a delayed closure of the aortic valve.
A 65-year-old male is brought into the emergency department after suddenly passing out on the street. The patient is now awake and alert. His vital signs are normal. On physical examination, a crescendo-decrescendo systolic murmur with ejection click is heard at the right upper sternal border, with radiation to the neck. The patient is immediately scheduled for an echocardiogram. How does this patient's cardiac cycle differ from normal? Select one: a. Decreased atrial filling b. Prolonged systolic ejection c. Decreased wall thickness d. Decreased isovolumic relaxation e. Decreased isovolumetric contraction
aortic regurgitation
A 66-year-old male is referred to a cardiologist for evaluation. Physical examination reveals a diastolic murmur prominent over the left sternal border, a decrease in diastolic pressure, and an increase in pulse pressure. Which of the following is the most likely diagnosis? Select one: a. Pulmonic stenosis b. Aortic stenosis c. Pulmonary regurgitation d. Mitral valve prolapse e. Aortic regurgitation
Aortic regurgitation
A 66-year-old male is referred to a cardiologist for evaluation. Physical examination reveals a diastolic murmur prominent over the left sternal border, a decrease in diastolic pressure, and an increase in pulse pressure. Which of the following is the most likely diagnosis? Select one: a. Aortic regurgitation b. Aortic stenosis c. Mitral valve prolapse d. Pulmonic stenosis e. Pulmonary regurgitation
An increased a wave The a wave is the increase in central venous pressure that normally occurs during atrial con- traction. Mitral regurgitation will produce a greater atrial preload and therefore a greater force of contraction. The increased atrial volume pro- duces an increased ventricular preload. Blood pressure is typically normal in patients with mitral regurgitation. The greater left ventricular preload produces a greater-than-normal stroke volume. However, the forward stroke volume, the volume entering the aorta, does not increase, so there is no increase in pulse pressure or cardiac output.
A 67-year-old man who has difficulty breathing when he exercises makes an appointment to see his physician. Auscultation reveals a holosystolic murmur leading to the diagnosis of mitral regurgitation. Which of the following laboratory findings is most likely to be present? Select one: a. An increased cardiac output b. An increased pulse pressure c. A decreased arterial pressure d. A decreased left ventricular preload e. An increased a wave
Infusion of a balanced electrolyte solution With severe diarrhea, there is a large loss of sodium and water from the body resulting in dehy- dration and sometimes shock. e best therapy is to replace the electrolytes that were lost during diar- rhea. erefore, infusion of a balanced electrolyte solution is the therapy of choice.
A 70-year-old man enters the hospital emergency department and has been experiencing severe diarrhea. He has pallor, tachycardia, and an arterial pressure of 80/50, and has trouble walking. Which of the following therapies would the physician recommend to prevent shock? Select one: a. Infusion of a sympathomimetic drug b. Administration of an antihistamine c. Blood infusion d. Administration of a glucocorticoid e. Infusion of a balanced electrolyte solution
3 Point (3) marks the end of systole and the beginning of diastole. he ventricles begin to relax, and part way through diastole the atria contract. Pressure in the left ventricle and aorta both begin to fall as blood travels through systemic circulation. Once the pressure in the ventricle falls below the pressure in the aorta, the aortic valve closes (3). This is where the murmur of aortic regurgitation begins. An incompetent aortic valve allows blood to flow backwards into the left ventricle through the aortic valve.
A 76-year-old male presents to his primary care physician with exertional dyspnea that has been worsening over the past four months. He is no longer able to do any household chores. He has also had swelling in his legs. He does not use pillows, but has been sleeping in a reclining chair for >10 years. He reports that he has no other medical problems, but that he has not seen a physician in more than 40 years. He does not take any medications. His family history is notable for a father with a bicuspid aorta. He has a 20-pack year smoking history and denies using any illicit drugs. Temperature 37°C (98.6°F), heart rate 80/min, blood pressure 132/68 mmHg, respiratory rate 18/min, and oxygen saturation 94% on room air. On physical exam, he bobs his head and has a pulsating uvula. He has a soft S1, an early diastolic decrescendo murmur at the right upper sternal border that increases with leaning forward, a diastolic rumble at the apex, and an S3. The PMI is diffuse and laterally displaced. There is a rapid rise and fall of the carotid pulses and crackles are noted in the bases of the lung bilaterally. The jugular venous pulse is approximately 10 cm and there is 1+ peripheral edema. A normal cardiac cycle is depicted in the diagram. The abnormal murmur in this patient begins at which numbered point of the cardiac cycle? Select one: a. 2 b. 1 c. 5 d. 6 e. 4 f. 3
the child could be the natural offspring of this couple There is no antigen and antibody reaction between the anti-B antibodies and the red cells. ere is a reac- tion between the anti-B antibodies and the red cells. erefore the red cells have the B antigen and must be type B. ere is no antigen and antibody reaction between the anti-D antibodies and the red cells. ere- fore the red cells must be Rh-negative. e child is blood type B- . Since father is AB- and the mother is B -, the child could be B-
A couple requests blood typing of their 2-year-old child (father AB, Rh-negative; mother B, Rh-negative). Results of hemagglutination assays of the child's blood are shown in the next figure. Which of the following conclusions concerning the child's parentage is valid? Select one: a. Neither the father nor the mother could be the natural parents b. the child could be the natural o ffspring of this couple c. the mother could be the natural mother, but the father could not be the natural father d. the father could be the natural father, but the mother could not be the natural mother
Arterial compliance The difference between systolic pressure and diastolic pressure is called the pulse pressure. e two main factors that affect pulse pressure are stroke volume and arterial compliance. Pulse pressure is directly proportional to the stroke volume and inversely pro- portional to the arterial compliance. us, a decrease in arterial compliance would tend to increase pulse pressure.
A decrease in which of the following tends to increase pulse pressure? Select one: a. Venous return b. Systolic pressure c. Stroke volume d. Arterial compliance e. Plasma volume
A positive
A lady presented with shock due to Post partum hemorrhage, her blood group was unknown but became coagulatedwhen mixed with serum containing anti-A antibodies and similarly her serum coagulated with B positive blood. The mostsuitable blood group to be transfused in this case is: Select one: a. AB positive b. A positive c. B positive d. O positive e. O negative
+105 Note that lead III has the strongest vector, there- fore the mean electrical axis will be closer to this lead than to leads I or II. e angle of lead III is 120°, and the resultant vector (mean electrical axis) is close to that lead and has a value of 105°.
A male long-term smoker who is 62 years old weighs 250 lb. He had the following EKG recorded at his local hospital. Which of the following is the mean electrical axis calculated from standard leads I, II, and III shown in his EKG? Select one: a. +105 b. +90 c. -20° d. -110° e. +180
150 beats/min The heart rate can be determined by 60 divided by the R-R interval, which gives you a value of 150 beats/ min. is is tachycardia, defined as a heart rate greater than 100 beats/min.
A male patient had a myocardial infarction at age 55. He is now 63 years old. Standard limb lead I is shown here. What is his heart rate? Select one: a. 40 beats/min b. 50 beats/min c. 100 beats/min d. 75 beats/min e. 150 beats/min
+10 Net filtration pressure (NFP) = (capillary hydrostatic pressure + interstitial colloid osmotic pressure) − (capillary colloid osmotic pressure + interstitial hydrostatic pressure). With the given values, interstitial colloid osmotic pressure equals 10 mm Hg, based on the following: 8 = (22 + X) − (17 + 7) = 22 + X − 24 = X − 2. When solved: X = 8 + 2 = 10
A medical student is studying fluid exchange in the skeletal muscle capillaries of a laboratory animal. He determines that fluid is being forced out of the capillaries with a net filtration pressure of 8 mm Hg, and he obtains the following laboratory values: Capillary hydrostatic pressure = 22 mm Hg Capillary colloid osmotic pressure = 17 mm Hg Interstitial hydrostatic pressure = 7 mm Hg Which of the following is the interstitial colloid osmotic pressure in mm Hg? Select one: a. - 6 b. +6 c. - 9 d. +8 e. +10
the child has no chance of developing Hemolytic disease of the newborn (HDN) HDN occurs when the mother is Rh- , the father is Rh+ , resulting in an Rh- child. Since the child is O- and the father is Rh -, there is no chance of HDN.
A mother of blood type A+ who has always been perfectly healthy just delivered her second child. !e father is of blood group O-. Knowing that the child is of blood group O- (O, Rh-), what would you expect to find in this child Select one: a. the child has no chance of developing Hemolytic disease of the newborn (HDN) b. the child will suffer from both A and B c. the child will suffer from erythroblastosis fetalis due to ABO blood group incompatibility d. the child will su ffer from erythroblastosis fetalis due to rhesus incompatibility
decreases
A negative inotropic effect is one that __________ myocardial contractility.
lying between 30 and 90 degrees
A normal QRS axis is typically defined as?
Phase 0: L-type calcium channels open, causing depolarization. Phases 1 and 2 are absent. Phase 3: Potassium channels open, causing repolarization. These channels can be affected by a number of pharmacologic mechanisms. Phase 4: Funny channels with a slow leak of potassium out of the cell cause a transient depolarization toward threshold.
A number of channels are involved in the cardiac action potential in the AV node. Describe what happens in each phase
The cardiac action potential passes through the AV node The PR interval starts at the beginning of the P wave and ends at the beginning of the QRS complex. The physiologic events that occur during this time period include atrial depolarization, which is responsible for the P wave, AV nodal depo- larization, and depolarization of the bundle of His and the Purkinje fibers. SA nodal depolarization precedes the P wave. Since the mass of the SA node is so small, this event cannot be detected on the standard ECG recording. The mitral and aortic valves are closed during isovolemic con- traction, which occurs after the QRS complex has begun. The second heart sound occurs at the end of systole.
A pacemaker is inserted in a patient in order to shorten the PR interval detected on their ECG. Which of the following events normally occurs during the PR interval? Select one: a. The ventricle is contracting b. There is no change in the voltage tracing on the ECG c. The second heart sound is heard d. The mitral and aortic valves are both closed e. The cardiac action potential passes through the AV node
C This diagram depicts cardiac (ventricular) function and venous function curves. With the onset of exercise, there is an increase in contractility, which shifts the cardiac function curve up. Changes often occur in tandem - exercise = inotropy+ decrease in TPR to maximize CO Also accompanying the onset of exercise are decreases in total peripheral resistance and venous compliance, both of which shift the vascular function curve to the right and increase its slope. The point at which the cardiac function and venous function curves inter- sect (C) represents the central venous pressure and cardiac output of the cardiovascular system under these conditions.
A patient is referred to the Heart Station for exercise stress testing. Baseline and exercise levels of cardiac and venous function are measured and plotted on the graphs below. The point marked "Control" represents baseline cardiovascular function in the resting state in the supine position. During treadmill exercise, there will be a shift from the resting state to which of the following points? Select one: a. A b. E c. B d. C e. D
Iron deficiency anemia the blood count values show that the patient is anemic. Her bone marrow is functioning and she has a normal platelet count, but is generating a decreased number of abnormal RBCs. e microcytic (small), hypochromic (decreased intracellular hemoglobin) is a classic description of iron deficiency anemia. With renal failure the patient would be anemic with normal RBCs. Sickle cell anemia has misshapen RBCs. Mega- loblastic anemia is characterized by macrocytic (large) RBCs.
A patient presents to your office complaining of extreme fatigue and shortness of breath on exertion that has gradually worsened over the past 2 weeks. On physical examination, you observe a well-nourished woman who appears comfortable but somewhat short of breath. Her vital signs include a pulse of 120, respiratory rate of 20,and blood pressure of 120/70. When she stands up her pulse increases to 150 and her blood pressure falls to 80/50. Her hematologic values are Hgb 7 g/dL, Hct 20%, RBC count 2 x 106/μl, platelet count of 400,000/μl. On a peripheral smear, her RBCs are microcytic and hypochromic. What would be your diagnosis of this patient? Select one: a. Sickle cell anemia b. Renal failure c. Megaloblastic anemia d. Iron deficiency anemia e. Aplastic anemia
Easily breakable clot Fibrin monomers polymerize to form a clot. To make a strong clot requires the presence of fibrin- stabilizing factor that is released from platelets within the clot. e other clotting tests determine the activa- tion of extrinsic and intrinsic pathways or number of platelets.
A patient suffers from a congenital deficiency in factor XIII (fibrin-stabilizing factor). What would analysis of his blood reveal? Select one: a. Prolonged whole blood clotting time b. Prolonged partial thromboplastin time c. Prolonged prothrombin time d. Easily breakable clot
nothing An Rh- mother will generate antibodies to the Rh+ red blood cells after the birth of the first child that is Rh+ . In the scenario presented, the mother has not been exposed to Rh+ RBCs so she has not devel- oped antibodies. However, after the birth of this child, and if the child is found to be Rh+, then anti-D immunoglobulin should be administered to the mother to destroy any fetal RBCs to which she has been exposed and to prevent her from forming antibodies to the Rh+ (D) antigen.
A pregnant woman comes in for a visit. She is AB Rh- and her husband is A Rh+ . this is her first child. What should be done at this time? Select one: a. Administer anti-D immunoglobulin to the mother at this time b. Nothing c. Administer anti-D immunoglobulin to the child if the child is Rh+ d. Administer anti-D immunoglobulin to the mother after delivery e. Administer anti-D immunoglobulin to the child after delivery
Intrinsic pathway the prothrombin time, test of extrinsic pathway, is the time required for clot formation following addition of tissue thromboplastin. is is normal so no problem with extrinsic. Partial thromboplastin time is a test of the intrinsic pathway. is is longer than normal so there is a problem with the intrinsic pathway. Bleeding time tests platelets, and since this is normal there is no problem with the platelets.
A teenaged boy with numerous nosebleeds was referred to a physician for evaluation prior to a minor surgery. His prothrombin time (PT) was 11 sec (11-15 sec normal), partial thromboplastin time (PTT) was 58 sec (25-40 sec normal), and bleeding time was 6.5 min (2-7 min normal). Which of the following is most likely abnormal in this young man? Select one: a. Extrinsic pathway b. Intrinsic pathway c. Production of clotting factors by the liver d. Decreased platelet number e. Defective platelets
aVF The positive portion of lead aVF has an axis of 90° and the negative part of this lead has an axis of 90°. Note the difference between the positive and the negative ends of this vector is 180°.
A ventricular depolarization wave when traveling ! 90° in the frontal plane will cause a large negative deflection in which lead? Select one: a. aVR b. aVL c. Lead II d. aVF e. Lead III
Pulmonary valve stenosis the mean electrical axis of the QRS of this patient is shifted rightward to 170°. "is indicates that the right side of the heart is involved. Both aortic stenosis and mitral regurgitation will cause a leftward shift of the QRS axis. Mitral stenosis will not affect the left ventricle but in severe enough circumstances could cause an increase in pulmonary artery pressure.this would cause an increase in pulmonary capillary pressure at the same time. Tricuspid stenosis will not a ffect the right ventricle. therefore, pulmonary valve stenosis is the only condition that fits this set of symptoms.
An 80-year-old male patient at UMC has been diagnosed with a heart murmur. Chest x-ray shows an enlarged heart but no edema fluid in the lungs. "e mean QRS axis of his EKG is 170°. Pulmonary wedge pressure is normal. What is the diagnosis? Select one: a. Mitral stenosis b. Mitral regurgitation c. Aortic stenosis d. Tricuspid stenosis e. Pulmonary valve stenosis
3.5 L/min Cardiac output can be measured by using Fick's principle, which asserts that the rate of uptake of a substance by the body (e.g., O2 consumption in milliliters per minute) is equal to the difference between its concentrations (milliliters per liter of blood) in arterial and venous blood multiplied by the rate of blood flow (cardiac output). This principle is restricted to situations in which arterial blood is the only source of the substance measured. If oxygen consumption by the body at steady state is measured over a period of time and the dif- ference in arterial O2 and venous O2 measured by sampling arterial blood and pulmonary arterial blood (which is fully mixed venous blood), cardiac output is obtained from the expression
An 82-year-old woman was admitted to the hospital with ascites, peripheral edema, and shortness of breath. Cardiac catheterization was ordered and the following values were obtained: Pulmonary artery O2 content = 20 mL/100 mLPulmonary vein O2 content = 12 mL/100 mLOxygen consumption = 280 mL/minStroke volume = 40 mL What is the woman's cardiac output? Select one: a. 8 mL/100 mL b. 3.5 L/min c. 4500 mL/min d. 7.0 L/min e. 2.86 L/min
An increase in the flow of sodium into the cell Phase-4 depolariza- tion is caused by the activation of a Na+ channel. The channel is activated when the membrane hyperpolarizes in contrast to the Na channel respon- sible for the action potential, which is activated when the cell depolarizes. Potassium conductance decreases during phase-4 depolarization and thus the flow of potassium out of the cell is diminished. However, this change in potassium current is not responsible for phase-4 depolarization. Chloride conductance does not change during phase 4. The Na/Ca exchanger main- tains low intracellular calcium at rest and may reverse its direction and pump calcium into the cell during phase 2 of the cardiac action potential. However, neither the Na/Ca exchanger nor the Na-K pump is involved in phase-4 depolarization.
An 84-year-old woman presents with paroxysmal dizziness, syncope, confusion, and fatigue. Her heart rate did not change when the patient was instructed to perform a Valsalva maneuver. 24-hour Holter monitoring revealed periodic episodes of sinus bradycardia. Phase-4 depolarization of SA nodal cells is caused by which of the following? Select one: a. An increase in the flow of sodium into the cell b. A decrease in the flow of chloride out of the cell c. An increase in the activity of the Na/Ca exchanger d. A decrease in the activity of the Na-K pump e. A decrease in the flow of potassium out of the cell
Reactive hyperemia
An increase in blood flow in response to or reacting to a prior period of decreased blood flow
increased cardiac output for a given end-diastolic volume An increase in contractility produces an increase in cardiac output for a given end-diastolic volume, or pressure. The Frank-Starling relationship demonstrates the matching of cardiac output (what leaves the heart) with venous return (what returns to the heart). An increase in contractility (positive inotropic effect) will shift the curve upward.
An increase in contractility is demonstrated on a Frank-Starling diagram by Select one: a. increased cardiac output for a given end-diastolic volume b. decreased cardiac output for a given end-diastolic volume c. increased cardiac output for a given end-systolic volume d. decreased cardiac output for a given end-systolic volume
Capillary wall hydraulic conductivity An increase in capillary wall permeability to water would increase capillary filtration rate, whereas increases in arteriolar resistance, plasma colloid osmotic pressure, and interstitial hydrostatic pressure would all decrease filtration rate. Plasma sodium concentration would have no effect on filtration.
An increase in which of the following tends to increase capillary filtration rate? Select one: a. Interstitial hydrostatic pressure b. Plasma sodium concentration c. Arteriolar resistance d. Plasma colloid osmotic pressure e. Capillary wall hydraulic conductivity
Hydraulic conductivity of the capillary wall and Capillary hydrostatic pressure The two main factors that increase lymph flow are an increase in capillary filtration rate and an increase in lymphatic pump activity. An increase in plasma colloid osmotic pressure decreases capillary filtration rate, interstitial volume and hydrostatic pressure, and lymph flow. In contrast, an increase in hydraulic con- ductivity of the capillary wall and capillary hydrostatic pressure increase capillary filtration rate, interstitial volume and pressure, and lymph flow. An increase arteriole resistance would decrease capillary hydrostatic pressure, capillary filtration rate, interstitial volume and pressure, and lymph flow.
An increase in which of the following would tend to increase lymph flow? Select one: a. Plasma colloid osmotic pressure b. Arteriolar resistance c. Hydraulic conductivity of the capillary wall and Capillary hydrostatic pressure d. Capillary hydrostatic pressure e. Hydraulic conductivity of the capillary wall
Point A
At what point in the Wiggers diagram below does the QRS complex begin? Select one: a. C b. D c. B d. E e. A
Renal artery
At which site is systolic blood pressure the highest? Select one: a. Renal artery b. Right atrium c. Renal vein d. Pulmonary artery e. Central vein f. Aorta
Physiologic splitting of S2
Because the aortic valve (A) closes slightly earlier than pulmonic valve (P), the seconds are sometimes heard separately (A2 then P2), more audible during inspiration due to relative lowering of the pressure in the pulmonary artery. This is known as?
E The diagram rep- resents the cardiac (ventricular) function and venous function curves. Similar to the familiar Starling curves, cardiac output is graphed as a func- tion of central venous pressure (preload). The cardiac function curves are shifted up and to the left by an increase in contractility and a decrease in afterload; they are shifted down and to the right by a decrease in contrac- tility and an increase in afterload. The vascular function curves graph cen- tral venous pressure as a function of cardiac output. The independent variable, cardiac output, is represented on the y axis. An increase in car- diac output causes a fall in central venous pressure. A decrease in blood volume or venous tone shifts the vascular function curves to the left; an increase in blood volume or venous tone shifts the vascular function curves to the right. The point at which the two curves intersect represents the central venous pressure and cardiac output of the cardiovascular sys- tem. The shift from the resting state to point E represents an increase in vascular volume or venous tone without any change in TPR or ventricular contractility. This is consistent with an infusion of saline or a blood transfusion. An increase in total peripheral resistance (TPR) decreases the slope; a decrease in TPR increases the slope. An increase in TPR decreases the slope of the vascular function curve, and the increase in contractility shifts the cardiac function curve up and to the right. The new intersection of these curves is at point A. The shift from the resting state to point B rep- resents an increase in contractility with no change in TPR. This is consis- tent with the administration of a positive inotropic drug. Upon sudden standing, contractility and TPR increase, whereas venous compliance decreases (i.e., venous tone increases). With exercise, there is an increase in contractility and a decrease in TPR and venous compliance.
Cardiac function and venous function curves were generated in a patient undergoing several maneuvers to evaluate his cardiac and cardiovascular reserves. Starting from the control point, to which point did the curves shift when the person was given a transfusion of saline? Select one: a. D b. C c. E d. B e. A
S2 heart sound
Closure of the semilunar valves (pulmonary and aortic)
Cardiac output decreases Loss of blood volume shifts the vascular function curve downward, resulting in an operating point at lower pressures and lower cardiac output.
Considering the vascular function curve, what would you expect to happen to cardiac output after a patient experiences hemorrhage. Ignore homeostasis reactions such as those from baroreceptors. Select one: a. Cardiac output increases b. Cardiac output stays the same c. Not enough information to answer d. Cardiac output decreases
decrease the intracellular [Ca2+].
Contractility is the ability to develop tension at a fixed muscle length. Factors that decrease contractility are those that ?
aortic pressure The electrocardiogram (ECG) tracing serves as a reference. The QRS complex marks ventricular depolarization, followed immediately by ventricular con- traction. Aortic pressure increases steeply after QRS, as blood is ejected from the ventri- cles. After reaching peak pressure, aortic pressure decreases as blood runs off into the arteries. The characteristic dicrotic notch ("blip" in the aortic pressure curve) appears when the aortic valve closes. Aortic pressure continues to decrease as blood flows out of the aorta.
Curve A in the figure represents Select one: a. ventricular volume b. aortic pressure c. ventricular pressure d. atrial pressure
ventricular volume Ventricular volume increases slightly with atrial systole (P wave), is constant during isovolumetric contraction (QRS), and then decreases dramatically after the QRS, when blood is ejected from the ventricle.
Curve B in the figure represents Select one: a. left atrial pressure b. atrial pressure c. ventricular pressure d. ventricular volume
Hereditary Spherocytosis
Defective spectrin and ankyrin •RBCs adopt a spherical shape and are osmotically fragile, prone to rupture, leading to Extravascular (in spleen) hemolysis, and anemia (reduced levels of circulating RBCs), aplastic crisis •Splenomegaly
D → X A normal Starling curve of the heart, showing stroke volume as a function of end-diastolic volume, is depicted by the curve on which an X is marked. The Starling curve below the normal one will result from a decrease in contractility (e.g., with heart failure) or an increase in afterload. The Starling curve above the normal curve will result from an increase in contractility or a decrease in afterload. Digitalis is a positive inotropic agent that will cause a point on the lower curve to shift to up and to the left, such that there is an increased stroke volume at a lower preload.
Digitalis is administered to a patient in congestive heart failure. Which of the shifts in the Starling curves shown below are consistent with the changes in ventricular function before and after digitalis in a patient with heart failure? Select one: a. C → B b. X → C c. X → A d. D → X e. E → D
Graph C Conduction abnormalities can produce first-degree, second-degree, or third-degree heart block. In a second-degree heart block, a P wave is not always followed by a QRS complex as in trace C, where the second P wave is not followed by a QRS complex. In a first-degree heart block, trace D, the interval between the beginning of the P wave and the beginning of the QRS complex (the PR interval) is longer than normal (greater than 0.2 seconds). In a third- degree heart block, conduction between the atria and ventricles is completely blocked so the atrial beats (represented by the P waves) and the ventricular beats (represented by the QRS complex) are completely dissociated.
During a routine physical examination, a 32-year-old female is found to have second-degree heart block. Which of the following ECG recordings was obtained from the patient during her physical examination? Select one: a. B b. C c. D d. A e. E
Liver Red blood cell production begins in the yolk sac for the first trimester. Production in the yolk sac decreases at the beginning of the second trimester and the liver becomes the predominate source of red cell production. During the third trimester red cell production increases from the bone marrow and con- tinues throughout life.
During the second trimester of pregnancy, where is the predominant site of red blood cell production? Select one: a. Lymph nodes b. Liver c. Yolk sac d. Bone marrow
Isovolumetric ventricular relaxation The mitral [atrioventricular (AV)] valve opens when left atrial pressure becomes higher than left ventricular pressure. This situation occurs when the left ventricular pressure is at its lowest level—when the ventricle is relaxed, blood has been ejected from the previous cycle, and before refilling has occurred.
During which phase of the cardiac cycle does the mitral valve open? Select one: a. Isovolumetric ventricular contraction b. Atrial systole c. Isovolumetric ventricular relaxation d. Rapid ventricular filling e. Rapid ventricular ejection f. Reduced ventricular filling (diastasis) g. Reduced ventricular ejection
ventricular ejection Aortic pressure reaches its highest level immediately after the rapid ejection of blood during left ventricular systole. This highest level actually coincides with the beginning of the reduced ventricular ejection phase.
During which phase of the cardiac cycle is aortic pressure highest? Select one: a. ventricular ejection b. Rapid ventricular filling c. Atrial systole d. Isovolumetric ventricular contraction e. Isovolumetric ventricular relaxation
Isovolumetric ventricular relaxation Ventricular volume is at its lowest value while the ventricle is relaxed (diastole), just before ventricular filling begins.
During which phase of the cardiac cycle is ventricular volume lowest? Select one: a. Isovolumetric ventricular contraction b. Rapid ventricular filling c. Isovolumetric ventricular relaxation d. Rapid ventricular ejection e. Atrial systole f. Reduced ventricular ejection g. Reduced ventricular filling (diastasis) Feedback
Hemophilia B
Factor IX deficiency
Fetal erythropoiesis
Fetal erythropoiesis occurs in: •Yolk sac (3-8 weeks of gestation) •Liver (6 weeks of gestation-birth) •Spleen (10-28 weeks of gestation) •Bone marrow (18 weeks of gestation to adult) mnemonic Young Liver Synthesizes Blood
β-Receptor antagonists work by inhibiting β-receptors, hindering the G-protein/cyclic adenosine monophosphate (cAMP) mechanism. By reducing the amount of cAMP and protein kinase A (PKA) produced, β-receptor antagonists decrease the Na+ and Ca2+ currents within the AV node. (Decreased production of cAMP and PKA reduces the Na+ current via phosphorylation-mediated downregulation of expression and other mechanisms). Thus β-receptor antagonists decrease the slope of phase 4 and phase 0. As a result, abnormal pacemakers can be suppressed. β-Receptor antagonists are known to slow the heart rate (negative chronotropic effect). β-Receptors do not utilize the inositol triphosphate mechanism and thus are not involved with protein kinase C. β-Receptor antagonists cause a decrease in cyclic adenosine monophosphate. β-Receptor antagonists cause a decrease in protein kinase A
Functions of β-receptor antagonists
A Histamine is a vasodilator that is typically released by mast cells and basophils. Infusion of histamine into a brachial artery would decrease arteriolar resistance and increase water permeability of the capillary wall. e decrease in arteriolar resistance would also increase capillary hydrostatic pressure. e increase in capillary hydrostatic pressure and water permeability leads to an increase in capillary filtration rate.
Histamine is infused into the brachial artery. Which of the following sets of microcirculatory changes would be expected in the infused arm? Select one: a. B b. A c. E d. F e. D f. H g. G h. C
ventricular septal defect
Holosystolic, harsh-sounding murmur Loudest at tricuspid area
heart rate = 300/N of large squares
How can you measure heart rate on ECG graph paper?
Active hyperemia
Illustrates the concept that blood flow to an organ is proportional to its metabolic activity
A
In the Wiggers diagram below, at which point will sarcomere length of left ventricular myocytes be greatest? Select one: a. E b. D c. A d. C e. B
point D The graph illustrates the development of pressure in the aorta, the left atrium, and the left ventricle during a single cardiac cycle. At point D, the pressure within the left ven- tricle is less than the pressure in the left atrium, and, therefore, the mitral valve opens and ventricular filling begins. Although the volume in the left ventricle is increasing, the pressure is falling. During this time period, the recoil of the ventricle causes its pressure to decrease as it is filling. Later in diastole, the pressure of the blood returning from the lungs causes both volume and pressure in the ventricle to increase.
In the hemodynamic pressure tracings below, rapid ventricular filling begins at which point on the figure below? Select one: a. B b. C c. A d. E e. D
Spinal cord injury Sympathomimetic drugs are given to counteract hypotension during a number of conditions. ese include spinal cord injury in which the sympathetic output is interrupted. Sympathomimetic drugs are also given during very deep anesthesia which decreases the sympathetic output, and in anaphylac- tic shock that results from histamine release and the accompanying vasodilatation. Sympathomimetic drugs, such as norepinephrine, increase blood pres- sure by causing a vasoconstriction. Shock caused by excess vomiting, hemorrhage or excessive adminis- tration of diuretics result in fluid volume depletion resulting in decreased blood volume and decreased mean systemic filling pressure. Giving a balanced electrolyte solution best counteracts this condition
In which of the following conditions will administration of a sympathomimetic drug be the therapy of choice to prevent shock? Select one: a. Spinal cord injury b. Hemorrhagic shock c. Shock caused by excess diuretics d. Shock due to excessive vomiting
Left-to-right ventricular shunt In a left-to-right ventricular shunt, a defect in the ventricular septum allows blood to flow from the left ventricle to the right ventricle instead of being ejected into the aorta. The "shunted" fraction of the left ventricular output is therefore added to the output of the right ventricle, making pulmonary blood flow (the cardiac BRS output of the right ventricle) higher than systemic blood flow (the cardiac output of the left ventricle). In normal adults, the outputs of both ventricles are equal in the steady state. In the fetus, pulmonary blood flow is near zero. Right ventricular failure results in decreased pulmonary blood flow. Administration of a positive inotropic agent should have the same effect on contractility and cardiac output in both ventricles.
In which of the following situations is pulmonary blood flow greater than aortic blood flow? Select one: a. Administration of a positive inotropic agent b. Right-to-left ventricular shunt c. Right ventricular failure d. Fetus e. Normal adult f. Left-to-right ventricular shunt
Septic shock In hemorrhagic shock, anaphylactic shock and neurogenic shock the venous return of blood to the heart markedly decreases. However, in septic shock the cardiac output increases in many patients because of vasodilation in affected tissues and by a high metabolic rate causing vasodilation in other parts of the body.
In which type of shock does cardiac output often increase? Select one: a. Septic shock b. Hemorrhagic shock c. Neurogenic shock d. Anaphylactic shock
left sided heart failure
Inability to pump blood to the systemic circulation -> ↑ LV pressure -> ↑ LVEDP -> ↑ LA pressure -> ↑ pulmonary capillary pressure (Pc) -> pulmonary edema -> dyspnea Orthopnea, Paroxysmal nocturnal dyspnea, Pulmonary edema •Finally lead to ↑ pressure in RV -> ↑ pressure in RA -> ↑ pressure in systemic veins (↑ jugular venous pressure) -> pitting edema
the aortic valve closes before the pulmonic valve The second heart sound is associated with closure of the aortic and pulmonic valves. Because the aortic valve closes before the pulmonic valve, the sound can be split by inspiration.
Inspiration "splits" the second heart sound because Select one: a. the mitral valve closes before the tricuspid valve b. filling of the ventricles has fast and slow components c. the tricuspid valve closes before the mitral valve d. the pulmonic valve closes before the aortic valve e. the aortic valve closes before the pulmonic valve
Folate deficiency
Macrocytic anemia: MCV > 100 fL, ↓ RBC count & Htc , Macrocitosis (Megaloblastic RBC)
Vitamin B12 deficiency
Macrocytic anemia: MCV > 100 fL, ↓ RBC count & Htc , Macrocitosis (Megaloblastic RBC), pernicious anemia, gastrectomy, , veganism, Diphyllobothrium latum, Neurologic symptoms)
iron deficiency
Microcytic anemia: MCV < 80 fL, ↓ RBC count & Htc , ↓ iron, ↓ ferritin, fatigue, conjunctival pallor, pica, spoon nails - koilonychia
Aplastic anemia
Normocytic (MCV = 80-100fL) suppression or destruction of the bone marrow , ↓ RBC count & Htc , ↓ reticulocyte count, ↑ EPO, leukopenia, and thrombocytopenia, Bone marrow with fatty infiltration
hemolytic anemia
Normocytic (MCV = 80-100fL) ↓ RBC count & Htc, increases in LDH, ↑ reticulocytes, ↑ unconjugated bilirubin, ↑ urobilinogen in urine
Iron deficiency anemia this patient is anemic and has low hemoglobin with small red cells. Vitamin B12 and folic acid defi- ciency will result in macrocytic red blood cells. His WBC and platelet counts are normal, suggesting a normal bone marrow. e positive stool shows a gas- trointestinal blood loss. A person can be anemic from a blood loss and have normal-sized RBCs as long as there is enough iron in the body. e microcytic and hypochromic RBCs are classic signs of iron deficient anemia.
Over the past 12 weeks, a 75-year-old man with a moderate aortic stenosis has developed shortness of breath and chest pains during exertion. He appears pale. Test of his stool for blood is positive. Laboratory studies show the following: hemoglobin 7.2 g/dL, and mean corpuscular volume 75. A blood smear shows microcytic, hypochromic erythrocytes. Which of the following is the most likely diagnosis? Select one: a. Autoimmune hemolytic anemia b. Folate deficiency anemia c. Iron deficiency anemia d. Vitamin B12 deficiency
atrial depolarization
P wave corresponds to what cardiac function?
Tricuspid regurgitation Early systolic murmurs begin with the first heart sound and end in mid- systole. The higher-than-normal height of the jugular blood column reflects an increased right atrial pressure. The combination of an early systolic mur- mur and high right atrial pressure is indicative of tricuspid regurgitation. This lesion is common in narcotic abusers with infective endocarditis. Mitral stenosis and aortic regurgitation produce diastolic murmurs.
Physical examination of a 41-year-old narcotic abusor reveals an early systolic murmur. The physician also notes a 7-cm distance between the height of the blood in his right internal jugular vein and sternal angle (normal = 3 cm). Which of the following conditions is most likely responsible for the physical findings? Select one: a. Tachycardia b. Tricuspid regurgitation c. Aortic regurgitation d. Atherosclerosis e. Mitral stenosis
determined by stroke volume Pulse pressure is the difference between the highest (systolic) and lowest (diastolic) arterial pressures. It reflects the volume ejected by the left ventricle (stroke volume). Pulse pressure increases when the capacitance of the arteries decreases, such as with aging.
Pulse pressure is Select one: a. the highest pressure measured in the arteries b. the lowest pressure measured in the arteries c. determined by stroke volume d. the difference between mean arterial pressure and central venous pressure e. measured only during diastole f. decreased when the capacitance of the arteries decreases
Decrease in Na+ and Ca2+ currents
Researchers are conducting a clinical study to examine the properties of a β-receptor antagonist (propranolol) in patients with supraventricular tachycardia. They find that the drug targets the atrioventricular (AV) node and lengthens the PR interval on an electrocardiogram (ECG). In addition to the decrease in AV conduction velocity, they observe that cardiac output is decreased. Which of the following describes the mechanism of action of the drug used in this trial? Select one: a. Decrease in inositol triphosphate b. Increase in cyclic adenosine monophosphate c. Decrease in protein kinase C d. Decrease in Na+ and Ca2+ currents e. Increase in protein kinase A
Fixed splitting
Seen in ASD. ASD causes left-to-right shunt = increased RA and RV causing volumes causing increased flow through pulmonic valve such that, regardless of breath, pulmonic closure is greatly delayed.
impulse conduction through the AV node
The PR segment corresponds to what cardiac function?
ventricular depolarization
The QRS complex corresponds to what cardiac function?
ventricular repolarization
The T wave corresponds to what cardiac function?
increased mean systemic pressure The shift in the venous return curve to the right is consistent with an increase in blood volume and, as a consequence, mean systemic pressure. Both cardiac output and venous return are increased in the new steady state (and are equal to each other). Contractility is unaffected
The change indicated by the dashed lines on the cardiac output/venous return curves shows Select one: a. decreased blood volume b. increased mean systemic pressure c. decreased venous return in the "new" steady state d. increased myocardial contractility e. decreased cardiac output in the "new" steady state
increased contractility An upward shift of the cardiac output curve is consistent with an increase in myocardial contractility; for any right atrial pressure (sarcomere length), the force of contraction is increased. Such a change causes an increase in stroke volume and cardiac output. Increased blood volume and increased mean systemic pressure are related and would cause a rightward shift in the venous return curve. A negative inotropic agent would cause a decrease in contractility and a downward shift of the cardiac output curve.
The dashed line in the figure illustrates the effect of Select one: a. increased contractility b. increased total peripheral resistance (TPR) c. a negative inotropic agent d. increased mean systemic pressure e. increased blood volume
E
The figure shows the aortic pressure, left ventricular pressure, and left atrial pressure during one cardiac cycle. Which of the following sets of data most closely corresponds to the figure? Select one: a. A b. E c. B d. C
0.05 mm Hg/mL/min The student must first recognize that the four organs are arranged in parallel The total resistance of the circuit (RT) is then calculated as follows 1/RT=1/0.2 +1/0.2 + 1/0.2 + 1/0.2=20 mm Hg/mL/min Therefore. RT=1/20=0.05 mm Hg/mL/min. The student should recall that adding resistances (R1, R2, R3...) in parallel reduces the total resistance (RT) of a circuit because of the manner in which parallel resistances are added, 1/RT=1/R1 + 1/R2 + 1/R3. Note also from the equation that removing a parallel resistance will increase the total resistance
The figure shows the vascular systems of four organs with resistances 1-4 The arterial inflow pressures and venous outflow pressures are the same for all organs Assuming that the resistance of the feed vessels is negligible, what is the total resistance of the entire circuit if the resistance of each of the four organs is 0.20 mm Hg/mL/min? Select one: a. 0.02 mm Hg/mL/min b. 0.05 mm Hg/mL/min c. 0.01 mmHg/mL/min d. 0.04 mm Hg/mL/min e. 20 mm Hg/mL/min
6.25 L/min Cardiac output is calculated by the Fick principle if whole body oxygen (O2) consumption and [O2] in the pulmonary artery and pulmonary vein are measured. Mixed venous blood could substitute for a pulmonary artery sample, and BRS Physiologyperipheral arterial blood could substitute for a pulmonary vein sample. Central venous pressure and heart rate are not needed for this calculation.
The following measurements were obtained in a male patient: Central venous pressure: 10 mm Hg Heart rate: 70 beats/min Pulmonary vein [O2] = 0.24 mL O2/mL Pulmonary artery [O2] = 0.16 mL O2/mL Whole body O2 consumption: 500 mL/min What is this patient's cardiac output? Select one: a. 4.55 L/min b. 6.25 L/min c. 1.65 L/min d. 5.00 L/min e. 8.00 L/min
6.25 L/min Cardiac output is calculated by the Fick principle if whole body oxygen (O2) consumption and [O2] in the pulmonary artery and pulmonary vein are measured. Mixed venous blood could substitute for a pulmonary artery sample, and BRS peripheral arterial blood could substitute for a pulmonary vein sample. Central venous pressure and heart rate are not needed for this calculation.
The following measurements were obtained in a male patient: Central venous pressure: 10 mm HgHeart rate: 70 beats/minPulmonary vein [O2] = 0.24 mL O2/mLPulmonary artery [O2] = 0.16 mL O2/mLWhole body O2 consumption: 500 mL/min What is this patient's cardiac output? Select one: a. 8.00 L/min b. 5.00 L/min c. 6.25 L/min d. 1.65 L/min e. 4.55 L/min
1:20
The graph below illustrates the pressure-volume curves for the arterial and venous systems. Which of the following is the approximate ratio of the arterial compliance to the venous compliance? Select one: a. 1:10 b. 15:1 c. 1:1 d. 10:1 e. 1:20
They have the greatest resistance The decrease in pressure at any level of the cardiovascular system is caused by the resistance of the blood vessels (∆P = Q × R). The greater the resistance is, the greater the decrease in pressure is. The arterioles are the site of highest resistance in the vasculature. The arterioles do not have the greatest surface area or cross-sectional area (the capillaries do). Velocity of blood flow is lowest in the capillaries, not in the arterioles.
The greatest pressure decrease in the circulation occurs across the arterioles because Select one: a. they have the greatest cross-sectional area b. they have the greatest surface area c. the velocity of blood flow through them is the lowest d. they have the greatest resistance e. the velocity of blood flow through them is the highest
Point D The plateau phase (phase 2) is the result of the influx of calcium. Although calcium channels begin to open during the upstroke (phase 0), the greatest number of cal- cium channels is open during the plateau. The upstroke is primarily depen- dent on the opening of Na+ channels. The initial repolarization (phase 1) is dependent on the inactivation of Na+ channels and the opening of a tran- sient K+ channel. Repolarization (phase 3) is produced by the inactivation of Ca2+ channels and the activation of the delayed rectifier K+ channels
The phases of the ventricular muscle action potential are represented by the lettered points on the diagram below. At which point on the ventricular action potential is membrane potential most dependent on calcium permeability? Select one: a. Point E b. Point A c. Point C d. Point B e. Point D
partial occlusion of a blood vessel Turbulent flow is predicted when the Reynolds number is increased. Factors that increase the Reynolds number and produce turbulent flow are decreased viscosity (hematocrit) and increased velocity. Partial occlusion of a blood vessel increases the Reynolds number (and turbulence) because the decrease in cross- sectional area results in increased blood velocity (v = Q/A).
The tendency for blood flow to be turbulent is increased by Select one: a. partial occlusion of a blood vessel b. increased viscosity c. increased hematocrit d. decreased velocity of blood flow
conduction system disturbances, inferior wall MI, and digitalis toxicity
Third-degree heart block is caused by?
hemolytic disease of the newborn •Rh: •Rh ⊝ mothers; Rh ⊕ fetus •First pregnancy: mother exposed to fetal blood (often during delivery) -> formation of maternal anti-D IgG. •Subsequent pregnancies: anti-D IgG crosses the placenta -> HDN in the fetus •Jaundice shortly after birth, kernicterus, hydrops fetalis •Prevent by administration of anti-D IgG to Rh⊝ pregnant women during third trimester and early postpartum period (if fetus tests ⊕ for Rh). Prevents maternal anti-D IgG production. •ABO: •A (B) fetus/O mother •Mother's anti-A (or anti-B) antibodies (IgG !!!) cross the placenta and agglutinize fetal RBCs •Can occur during 1st pregnancy
This disease occurs in the fetus if the fetus is Rh+ while the mother is Rh-.
disorders of vascular tone or blood volume, cardiovascular disorders, or cerebrovascular disease. Approximately one-fourth of syncopal episodes are of cardiac origin
Transient decreases in cerebral blood flow are usually due to one of three general mechanisms:
500 ml/min Blood flow in a vessel is directly proportional to the fourth power of the vessel radius. Increasing vesseldiameter by 50% (1.5 times control) would increase blood flow 1.5 to the fourth power % normal blood flow (100 ml/min). " us, blood flow would increase to 100 ml/min % 5.06, or approximately 500 ml/min.
Under control conditions, flow through a blood vessel is 100 ml/min with a pressure gradient of 50 mm Hg.What would be the approximate flow through the vessel after increasing the vessel diameter by 50%, assuming the pressure gradient is maintained at 100 mm Hg? Select one: a. 500 ml/min b. 300 ml/min c. 150 ml/min d. 100 ml/min e. 700 ml/min
increased after load
Ventricular pressure must rise to a greater-than-normal
Bile duct obstruction Most clotting factors are formed in the liver. If a vitamin K injection will correct the problem, this implies that the liver is working fine, and that the patient does not have hepatitis. Vitamin K is a fat-soluble vitamin and is absorbed from the intestine along with fats. Bile secreted by the gallbladder is required for the absorption of fats. If the patient is deficient in vitamin K, then clotting deficiency can be corrected by an injection of vitamin K. Antithrombin II has no relationship to factor IX.
What condition leads to a deficiency in factor IX that can be corrected by an intravenous injection of vitamin K? Select one: a. Classic hemophilia b. Bile duct obstruction c. Genetic deficiency in antithrombin III d. Hepatitis B
Increasing heart rate increases intracellular [Ca2+] because more Ca2+ ions enter the cell during the plateau of each action potential. Sympathetic stimulation and norepinephrine increase intracellular [Ca2+] by increasing entry during the plateau and increasing the storage of Ca2+ by the sarcoplasmic reticulum (SR) [for later release]. Cardiac glycosides increase intracellular [Ca2+] by inhibiting the Na+-K+ pump, thereby inhibiting Na+-Ca2+ exchange (a mechanism that pumps Ca2+ out of the cell).
What increases intracellular Ca2+
70 The heart rate can be calculated by 60 divided by the R-R interval, which is 0.86 sec. is results in a heart rate of 70 beats/min.
What is the heart rate in the following EKG? Select one: a. 104 b. 88 c. 94 d. 64 e. 70
-40 mV The normal resting membrane potential of the S-A node is 55 mV. As the sodium leaks into the mem- brane an upward drift of the membrane potential occurs until it reaches 40 mV. is is the threshold level that initiates the action potential at the S-A node
What is the membrane potential (threshold level) at which the S-A node discharges? Select one: a. -105 mV b. -55 mV c. -65 mV d. -85 mV e. -40 mV
Tissue trauma, formation of prothrombin activator, conversion of prothrombin into thrombin, conversion of fibrinogen into fibrin threads the extrinsic pathway involves damage to the tis- sue, then the subsequent formation of prothrombin activator. Tissue trauma results in the release of tissue factor or tissue thromboplastin, which functions as a proteolytic enzyme. Tissue factor binding with factor VII results in an activation of factor X. ere is a subsequent activation of prothrombin activator, con- version of prothrombin to thrombin, then the conver- sion of fibrinogen into fibrin threads. Activation of the extrinsic pathway is very fast because of the small number of enzymatic reactions.
What is the proper pathway for the extrinsic clotting pathway? Select one: a. Trauma to the blood, formation of prothrombin activator, conversion of prothrombin into thrombin, conversion of fibrinogen into fibrin threads b. Activation of platelets, formation of prothrombin activator, conversion of prothrombin into thrombin, conversion of fibrinogen into fibrin threads c. Contact of blood with collagen, formation of prothrombin activator, conversion of prothrombin into thrombin, conversion of fibrinogen into fibrin threads d. Tissue trauma, formation of prothrombin activator, conversion of prothrombin into thrombin, conversion of fibrinogen into fibrin threads
-55 mV The resting membrane potential of the sinus nodal fibers is 55 mV, and this is in contrast with the 85 to 90 mV membrane potential of cardiac muscle. Other major differences between the sinus nodal fibers and ventricular muscle fibers are that the sinus fibers exhibit self-excitation from inward leaking of sodium ions.
What is the resting membrane potential of the sinus nodal fibers? Select one: a. -80 mV b. -55 mV c. -100 mV d. -90 mV e. -20 mV
left arm By convention, the left arm is the positive elec- trode for lead I of an EKG
When recording lead I on an EKG, the right arm is the negative electrode, and the positive electrode is the Select one: a. left leg b. right arm + left leg c. right leg d. left arm e. left arm + left leg
left leg
When recording lead II on an EKG, the positive electrode is the Select one: a. right arm + left leg b. left arm c. right leg d. left leg e. left arm + left leg
left arm By convention, the left arm is the positive elec- trode for lead aVL of an EKG
When recording lead aVL on an EKG, the positive electrode is the Select one: a. right arm + left leg b. left arm + left leg c. right leg d. left arm e. left leg
A there is an antigen and antibody reaction between the anti-A antibodies and the red cells. ere is no reaction between the anti-B antibodies and the red cells. erefore the red cells have the A antigen and the cells must be type A.
Which blood type is depicted in the following figure? Select one: a. A b. AB c. O d. B
A Resistance of a vessel = pressure gradient / blood flow of the vessel. In this example, vessel A has the highest vascular resistance (100 mm Hg/1000 ml/min, or 0.1 mm Hg/ml/min).
Which blood vessel has the highest vascular resistance? Select one: a. C b. D c. A d. B e. E
Acetylcholine (ACh) Acetylcholine (ACh) has a negative inotropic effect on the atria.
Which of the following agents or changes has a negative inotropic effect on the heart? Select one: a. Norepinephrine b. Acetylcholine (ACh) c. Sympathetic stimulation d. Cardiac glycosides e. Increased heart rate
Type O, Rh +ive packed red cells
Which of the following blood units carries the least risk of inducing an immediate reaction in a type B, Rh+ patient? Select one: a. Type A , Rh+ive whole blood. b. Type AB, Rh -ive packed red cells c. Type O, Rh +ive packed red cells d. Type O, Rh +ive whole blood e. Type AB, Rh +ive whole blood.
Packed red cells O+ In any patient, transfusion of O-type packed cells will minimize a transfusion reaction since the antibodies will be removed with the plasma removal. If the Rh fac- tor is matched, then this will also minimize transfusion reaction. erefore, in a B patient, a B transfusion or an O transfusion will elicit no transfusion reaction.
Which of the following blood units carries the least risks for inducing an immediate transfusion reaction into a B$ (B, rhesus positive) recipient? Select one: a. Whole blood AB+ b. Whole blood A+ c. Packed red cells O+ d. Packed red cells AB- e. Whole blood O+
Veins THe percentage of total blood volume in the veins is approximately 64%.
Which of the following components of the circulatory system contains the largest percentage of the total blood volume? Select one: a. Arteries b. Capillaries c. Pulmonary circulation d. Veins e. Heart
Pulmonary hypertension Systemic hypertension results in a left axis devia- tion because of the enlargement of the left ventricle. Aortic valve stenosis and aortic valve regurgitation also result in a large left ventricle and left axis devia- tion. Excessive abdominal fat, because of the mechani- cal pressure of the fat, causes a rotation of the heart to the left resulting in a leftward shift of the mean electri- cal axis. Pulmonary hypertension causes enlargement of the right heart and thus causes right axis deviation.
Which of the following conditions will usually result in right axis deviation in an EKG? Select one: a. Pulmonary hypertension b. Excess abdominal fat c. Aortic valve regurgitation d. Systemic hypertension e. Aortic valve stenosis
the father of the child has to be Rh$ HDN occurs when an Rh mother gives birth to a second Rh child. erefore, the father has to be Rh . e mother becomes sensitized to the Rh anti- gens following the birth of the first Rh child. HDN is prevented by treating the mother with antibodies against Rh antigen after the birth of each Rh child. is will destroy all fetal RBCs in the mother and pre- vent the mother from being sensitized to the Rh anti- gen. A complete blood transfusion of the mother would be required to prevent the formation of Rh antibodies, but this is impractical. A transfusion of the first child after the birth will not accomplish anything as the mother has been exposed to the Rh antigen during the birth process.
Which of the following is TRUE concerning erythroblastosis fetalis (hemolytic disease of the newborn, HDN)? Select one: a. the father of the child has to be Rh$ b. this is prevented by giving the mother a blood transfusion c. A complete blood transfusion after the first birth will prevent HDN d. this occurs when a Rh$ mother has an Rh# child
Increased blood volume Venous return of the heart is equal to the mean systemic filling pressure minus the right atrial pressure divided by the resistance to venous return. erefore, decreased mean systemic filling pressure will decrease the venous return to the heart. Factors that will decrease the systemic filling pressure include large vein dilation, decreased sympathetic tone, increased venous compliance and increased blood volume.
Which of the following is normally associated with an increased venous return of blood to the heart? Select one: a. Acute large vein dilation b. Decreased mean systemic filling pressure c. Increased blood volume d. Decreased sympathetic tone e. Increased venous compliance
Increased blood volume Venous return of the heart is equal to the mean systemic filling pressure minus the right atrial pressure divided by the resistance to venous return. erefore, decreased mean systemic filling pressure will decrease the venous return to the heart. Factors that will decrease the systemic filling pressure include large vein dilation, decreased sympathetic tone, increased venous compliance and decreased blood volume.
Which of the following is normally associated with an increased venous return of blood to the heart? Select one: a. Increased blood volume b. Acute large vein dilation c. Increased venous compliance d. Decreased mean systemic filling pressure e. Decreased sympathetic tone
Veins The vascular compliance is proportional to the vascular distensibility and vascular volume of any given segment of the circulation. e compliance of a systemic vein is 24 times that of its corresponding artery because it is about 8 times as distensible and it has a volume about 3 times as great.
Which of the following parts of the circulation has the highest compliance? Select one: a. Veins b. Large arteries c. Capillaries d. Aorta e. Small arteries
aorta THe velocity of blood flow within each segment of the circulatory system is inversely proportional to the total cross-sectional area of the segment. Because the aorta has the smallest total cross-sectional area of all circulatory segments, it has the highest velocity of blood flow.
Which of the following segments of the circulatory system has the highest velocity of blood flow? Select one: a. Veins b. Venules c. Aorta d. Arteries e. Capillaries
Glucose Because O2, CO2, and CO are lipophilic, they cross capillary walls primarily by diffusion through the endothelial cell membranes. Glucose is water- soluble; it cannot cross through the lipid component of the cell membrane and is restrict- ed to the water-filled clefts, or pores, between the cells.
Which of the following substances crosses capillary walls primarily through water-filled clefts between the endothelial cells? Select one: a. CO2 b. CO c. Glucose d. O2
A Rh- whole blood to a B Rh- patient Transfusion of Rh blood into a Rh person with the same ABO type will not result in any reaction. Type A blood has A antigen on the surface and type B antibodies. Type B blood has B antigens and A anti- bodies. erefore, transfusing A blood into a person with type B blood will cause the A antibodies in the type B person to react with the donor blood.
Which of the following transfusions will result in an immediate transfusion reaction? Select one: a. O Rh+ whole blood to an O Rh- patient b. AB Rh- whole blood to an AB Rh+ patient c. A Rh- whole blood to a B Rh- patient d. B Rh whole blood to an B Rh- patient
Capillaries The capillaries have the largest total cross- sectional area of all vessels of the circulatory system. e venules also have a relatively large total cross- sectional area, but not as great as the capillaries, which explains the large storage of blood in the venous sys- tem compared with that in the arterial system.
Which of the following vessels has the greatest total cross-sectional area in the circulatory system? Select one: a. Small arteries b. Venules c. Capillaries d. Aorta e. Vena cava
Type A Rh+ packed cells to an O Rh+ patient Type O RBCs are considered to be universal donor blood. Reactions occur between the recipient's antibody and donor antigen as shown in the following table.
Which of the following will result in a transfusion reaction? Assume that the patient has never had a transfusion. Select one: a. Type A Rh+ packed cells to an O Rh+ patient b. Type AB Rh+ packed cells to an AB Rh+ patient c. Type A Rh+ packed cells to an A Rh- patient d. Type O Rh- packed cells to an AB Rh+ patient
Movement of leg muscles Interstitial hydrostatic pressure in a muscle capil- lary bed is normally negative ( 3 mm Hg). Pumping by the lymphatic system is the basic cause of the nega- tive pressure.
Which of the following would decrease venous hydrostatic pressure in the legs? Select one: a. Presence of ascitic fluid in the abdomen b. Pregnancy c. Increase in right atrial pressure d. Movement of leg muscles
Aspirin Heparin is used for the prevention of a clot, but has to be infused. is occurs by binding to antithrom- bin III and the subsequent inactivation of thrombin. Warfarin is used to inhibit the formation of vitamin K clotting factors. Aspirin is used to prevent activation of platelets. Activation of platelets following exposure to an atherosclerotic plaque and the formation of a platelet plug will impede blood flow and result in an ischemic heart attack. Streptokinase is used to break down an already formed clot, which is appropriate therapy for a pulmonary embolus
Which of the following would most likely be used for prophylaxis of transient ischemic heart attack? Select one: a. Streptokinase b. Aspirin c. Heparin d. Warfarin
Aspirin Heparin is used for the prevention of a clot, but has to be infused. is occurs by binding to antithrom- bin III and the subsequent inactivation of thrombin. Warfarin is used to inhibit the formation of vitamin K clotting factors. Aspirin is used to prevent activation of platelets. Activation of platelets following exposure to an atherosclerotic plaque and the formation of a platelet plug will impede blood flow and result in an ischemic heart attack. Streptokinase is used to break down an already formed clot, which is appropriate therapy for a pulmonary embolus
Which of the following would most likely be used for prophylaxis of transient ischemic heart attack? Select one: a. Streptokinase b. Aspirin c. Warfarin d. Heparin
E Aplastic anemia is a condition in which the bone marrow has a decreased production but does not respond to erythropoietin. erefore, a person with aplastic anemia would have a low hematocrit and an elevated erythropoietin level.
Which points in the following graph most closely define Aplastic anemia? Normal erythropoietin (EPO) levels are approximately 10. Select one: a. A b. E c. D d. C e. B
D A well-trained athlete will have a slightly elevated EPO level and the hematocrit will be elevated up to a value of 50%. A hematocrit higher than 50% suggests EPO treatment.
Which points in the following graph most closely define Olympic marathoner? Normal erythropoietin (EPO) levels are approximately 10. Select one: a. C b. D c. B d. A e. E
α1 Receptors The α1 receptors for norepinephrine are excitatory on vascular smooth muscle and cause vasoconstriction. There are also β2 receptors on the arterioles of skeletal muscle, but they produce vasodilation.
Which receptor mediates constriction of arteriolar smooth muscle? Select one: a. b1 Receptors b. b2 Receptors c. a1 Receptors d. Muscarinic receptors
Vitamin K deficiency the clotting factors are formed in the liver and require vitamin K. Vitamin K is a fat-soluble vitamin and absorption is dependent on adequate fat digestion and absorption. erefore, any state of malnutrition would have a decreased fat absorption, and results in decreased vitamin K absorption and decreased syn- thesis of clotting factors.
Why do some malnourished patients bleed excessively when injured? Select one: a. Platelet sequestration by fatty liver b. Serum bilirubin raises neutralizing thrombin c. Low serum-protein levels cause factor XIII problems d. Vitamin K deficiency
Sinus arrhythmia
a change of the heart rate produced by the normal variation in the rate of phase 4 depolarization of the SA nodal pacemaker cells between inspiration and expiration?
Hemophilia
a genetic loss of clotting factor VIII
Tachycardia
a heart rate above 100 beats per minute
Syncope (fainting)
a transient loss of consciousness caused by an inadequate blood flow to the brain.
Sinus Bradycardia
characterized by normal sinus rhythm, with a P wave before every QRS, and a ventricular rate < 60 beats per minute (bpm). This can be normal for some individuals or can result from medications like calcium channel blockers or beta blockers. Some individuals may be asymptomatic if their baseline is < 60 bpm, while others may develop dizziness, syncope, and hypotension.
Mobitz type I
characterized by progressive PR interval lengthening, until a beat is "dropped," indicating that the depolarization signal failed to pass from the atria to the ventricles. It is usually asymptomatic and is often caused by medications such as digoxin, calcium channel blockers, and beta-blockers. It can also be caused by right-sided heart ischemia. It is treated by stopping the medication, although atropine is sometimes used
S1 heart sound
closure of AV valves (mitral and tricuspid)
The QR segment
corresponds to ventricular depolarization, which is Phase 0 of the myocyte action potential and is characterized by sodium influx.